Saturday, June 14, 2014

The AFR, the Fallacy of Composition, and Underdetermination

A lot of anti-AFR arguments make the charge that it commits the fallacy of composition. My claim is that sometimes the status of the upper-level state follows necessarily from the base-level states, and sometimes it leaves the hopelessly underdetermined. The state of a planet seems to be fully determined by the states of the particles that make up the planet (plus relations to the star it revolves around, etc.) Mental states are, as I see it, hopelessly underdetermined by the physical. The physical states are compatible with numerous mental states, or even a complete lack of mental states entirely. 

177 comments:

Ilíon said...

Anti-AfR "arguments" always boil down to "How DARE you refuse to accept my question-begging as being valid reasoning!"

Unknown said...

I've never seen anyone say that the AfR commits the fallacy of composition. In fact, it doesn't even make sense to say so: it would seem to me that if the mental is really the effect of a "welling-up" (to borrow a picture from Daniel Dennett) of physical causes, then the property of physicalness is distributable from the parts to the whole, because the mental is really just a physical effect. So the AfR avoids committing the fallacy, or so it seems to me.

jdhuey said...

To be frank, I can't say that I have ever really understood how the AFR is suppose to show that the mental can't be based on physical processes.

Information, in certain technical senses, is part and parcel of physics. There is a deep and profound interconnection between physical reality and information. I can think of no circumstances where there could be information and for it NOT to be encoded onto something real (clay tablets, flash drives, beams of electromagnetic radiation, brains, etc.).

As I see it, the physical processes of the body and brain encode a vast amount of information and that information is processed in the various parts of the brain. In order to use the results of that mental processing for some purposes we integrate the information in a very complicated interconnected feedback process that is the experience of consciousness. At no point does anything happen that is not consistent with the natural world and there is no need to invoke some external supernatural factor.

Given the above exactly how does the AFR show that the mental is somehow inconsistent with naturalism?

B. Prokop said...

"I can't say that I have ever really understood how the AFR is suppose to show that the mental can't be based on physical processes.

The AFR does not say that "the mental can't be based on [solely] physical processes" - what it does say is that, if that is the case, then we have no reason to trust the results (of our mental processes).

By the way, I add the "[solely]" to your quote because the AFR does not exclude involvement of the physical, it simply states that that cannot be the "end of the story".

jdhuey said...

"Mental states are, as I see it, hopelessly underdetermined by the physical. The physical states are compatible with numerous mental states,..."

Parallel with what I wrote above, given that the physical can encode information how can you say that the mental states are underdetermined?

jdhuey said...

"...then we have no reason to trust the results (of our mental processes)."

Again, I don't fully understand how you can get to that conclusion. We know that our mental processes are somewhat reliable because we are not dead yet. And we know that our mental processes are somewhat unreliable because we get fooled by illusions frequently. So, given that, why can't the physical be "the end of the story" ?

amorbis said...

jdhuey,

"Information" is not something that objectively exists "encoded" in physical processes. Information only counts as information insofar as a mind interprets it as such. The words you see on the screen "encode" information in a figurative sense, but intrinsically they are nothing more than meaningless pixels. If you think that "information" is a real, objective part of reality, independent of any mind, then you have ceased to be a materialist and have admitted the existence of irreducible intentionality ("about"-ness) in nature.

jdhuey said...

Amorbis,

There is a deep and powerful interconnection between information and entropy - see http://en.wikipedia.org/wiki/Entropy_(information_theory).

In this context information has a very technical definition that is not to be confused with what we would call 'meaning'. So, given that one of the characteristics of life is that it is a local decrease in entropy, it is not at all surprising that information processing is integral to life.

amorbis said...

jdhuey,

The context does not help. Any notion of "information" will necessarily be infused with intentionality, and thus cannot be used to explain the mind in "naturalistic" terms.

im-skeptical said...

DNA contains physical information, and it has no need for any human or rational understanding in order to play a role in the specification of development of embryos.

jdhuey said...

Amorbis,

"The context does not help. Any notion of "information" will necessarily be infused with intentionality, and thus cannot be used to explain the mind in "naturalistic" terms.

This confuses me. Let's assume that the world is devoid of any multi-cellular creatures - let's say there is nothing but bacteria amoebas running around. Now when a bacterium gets near an amoeba there is a chemical signal that alerts the amoeba and it starts a chemical cascade that results in a pseudo-pod being extended toward the tasty little bacterium and envelops it. Now to my mind, this is just a case of information processing that enables the amoeba to maintain a local decrease in entropy at the expense of greatly increasing the entropy elsewhere.

So, can you tell me where the "irreducible intentionality" comes in?

amorbis said...

jdhuey,

The concept of "information-processing" does not need to be invoked in order to explain what happens in the situation you described. Telling a story about the laws of physics and the causal relationship between cells/molecules/atoms/particles will do just fine. You can describe it in information-processing terms if that results in a more helpful explanation, but that is something that we project onto it with our minds; it is not something that is forced on us by being there objectively. So no, there is no irreducible intentionality in your example - but neither is there any case of "information-processing" in any literal, objective sense.

Ilíon said...

"The AFR does not say that "the mental can't be based on [solely] physical processes" - what it does say is that, if that is the case, then we have no reason to trust the results (of our mental processes)."

It's my "stronger" AfR-like argument (sketched here) which says that "the mental can't be based on [solely] physical processes" -- though, to be more precise, "physical processes" are only a specific example or instance of what my argument shows to be impossible to be "the basis" of "the mental"; the general category being "not-mind" (*), which includes both "deterministic processes" and "random events".


(*) after all, *nothing* is "the basis" of "the mental" except "the mind", and "the mind" is not reducible to any "lower" entity.

Ilíon said...

jdhuey: "To be frank, I can't say that I have ever really understood how the AFR is suppose to show that the mental can't be based on physical processes.

Information, in certain technical senses, is part and parcel of physics ...
"

Sure, in thoise "technical senses" by which one misuses the concept we term 'information', and the term.

jdhuey: "I can think of no circumstances where there could be information and for it NOT to be encoded onto something real ..."

Ah! So, "personal incredulity" *is* dispositive ... when it needs to be?

jdhuey: "There is a deep and profound interconnection between physical reality and information. I can think of no circumstances where there could be information and for it NOT to be encoded onto something real (clay tablets, flash drives, beams of electromagnetic radiation, brains, etc.)."

In the famous phrase, you're consusing the map for the territory; you're making the category error of conflating the representation of information with that information.

You're also begging the question: you're starting with the (false) assumption that only physical entities are real.

jdhuey: "(clay tablets, flash drives, beams of electromagnetic radiation, brains, etc.)."

And there is a second category error in what you need to believe to be the truth about the nature of reality: "clay tablets, flash drives, beams of [complex specified] electromagnetic radiation" on the one hand, and "brains" on the other.

The physical representation of information by means of inherently meaningless physical entities -- symbols -- does nothing toward making there be "a deep and profound interconnection between physical reality and information".

Moreover, your very choice of words gives a clue to your error -- "[there are] no circumstances where there could be information and for it NOT to be encoded onto something real". To put it another way, what you're saying is "[there are] no circumstances where there could be information and for it NOT to be [represented by inherently meaningless physical entities which have utterly no relationship, neither logical nor conceptual, to the information so represented]"

Sie verstehen nicht, dass was Sie haben gesagt! -- if the false thing you want-and-need to believe were true, then *everyone* could understand that little sentence, whether they've ever been taught German ... or to read.

At the same time, were the false thing you need to be beleive to be true actually true, then there couldn't be multiple languages amongst men; perhaps not even *any* language. What is the word that denotes, say 'cat', when the word *is* the information? What is the word that denotes *both* 'Fluffy' and 'Tabby' when the word *is* the information?

Ilíon said...

I-pretent-at-reason: "DNA contains physical information, and it has no need for any human or rational understanding in order to play a role in the specification of development of embryos."

Is there anything *more* humorous that seeing a rabid atheist-and-DarwinDefender turn pro-ID so as to try to protect reductive/eliminative 'materialism' (as though there were some other kind) from ratonal critical evaluation?

Ilíon said...

amorbis: ""Information" is not something that objectively exists "encoded" in physical processes. Information only counts as information insofar as a mind interprets it as such. The words you see on the screen "encode" information in a figurative sense, but intrinsically they are nothing more than meaningless pixels. If you think that "information" is a real, objective part of reality, independent of any mind, then you have ceased to be a materialist and have admitted the existence of irreducible intentionality ("about"-ness) in nature."

Exactly ... well, other than this sentence, "Information only counts as information insofar as a mind interprets it as such", which is more correctly stated in two separate (and distinct) statements:
1) 'information' exists only insofar as some mind knows it;
2) the representation of information by means of symbols -- inherently meaningless (i.e. not-information) arbitrary physical entities -- counts *as* a representation of said information only insofar as at least one mind arbitrarily ascribes some specific meaning to the symbols.

amorbis: "The words you see on the screen "encode" information in a figurative sense, but intrinsically they are nothing more than meaningless pixels."

But, yes, the information conveyed by the physical symols is not in the symbols, nor attached to them, nor related to them in any manner whatsoever.

B. Prokop said...

"Is there anything *more* humorous that seeing a rabid atheist-and-DarwinDefender turn pro-ID so as to try to protect reductive/eliminative 'materialism' (as though there were some other kind) from ratonal critical evaluation?"

I was thinking the same thing, and quite literally LOL-ing over seeing Skep embracing ID in order to save his materialism. Quite the game there.

im-skeptical said...

The fact that there is no intelligence or rationality involved is what you see as ID? Now that's amusing.

World of Facts said...

The main problem with the AfR is that the proponent always start with the notion that minds can and do exist without a body, and then reach some in-coherency when trying to explain the mind with physical terms, hence concluding that the mind is not physical. The conclusion is implied first and reached second. This line by Mr. Prokop is 1 example of such implied thinking:
"what it does say is that, if that is the case, then we have no reason to trust the results (of our mental processes)"

B. Prokop said...

"The fact that there is no intelligence or rationality involved is what you see as ID? Now that's amusing."

No, what is amusing is the lack of intelligence or rationality in your postings.

B. Prokop said...

Upon reflection, I retract my last comment. Skep's postings are in no way "amusing - they are sad and somewhat depressing. And that's when they're not downright hate-filled and slanderous. And Skep doesn't even have a good excuse for acting as he does. Linton at least is (most likely) demon possessed, and therefore not entirely responsible for his behavior (or, should I say "behaviour"?). Skep has no such excuse to fall back on.

im-skeptical said...

Sorry, Hugo. It was never my intent to derail the discussion. It just seems to happen. Of course, you make a very good point, which has been completely ignored, in favor of the usual barrage of insults and inane comments about my lack of intelligence, etc. I would like to see substantive responses to the issues raised here, but I'm afraid they don't have any.

im-skeptical said...

Ilion believes he has an iron-clad case for the incoherence of materialism. Let's take a look:

"IF atheism were indeed the truth about the nature of reality, THEN this movement from (what we call) thought to though (which activity or change-of-mental-state we call 'reasoning') *has* to be caused by, and must be wholely explicable in terms of, state-changes of matter. That is, it is not the content of, and logical relationship between, two thoughts which prompts a reasoning entity to move from the one thought to the other, but rather it is some change-of-state of some matter which determines that an entity "thinks" any particular "thought" when it does."

The implicit assumption here is that logical processes cannot be mechanical, but must instead be accomplished by some kind of homuncular entity residing within us. This is circular reasoning. The assumption is the same as the conclusion. I honestly don't see how any dualist can think they have made a valid argument, when the underlying assumption of the separate or immaterial rational being is ALWAYS at the heart of these arguments. Ilion prides himself on his impeccable logic. Why can't he see the circularity of his argument?

Ilíon said...

hugely erroneous: "The main problem with the AfR is that the proponent always start with the notion that minds can and do exist without a body, and then reach some in-coherency when trying to explain the mind with physical terms, hence concluding that the mind is not physical. The conclusion is implied first and reached second. This line by Mr. Prokop is 1 example of such implied thinking:
"what it does say is that, if that is the case, then we have no reason to trust the results (of our mental processes)"
"

God-haters have no shame; they will say *anything* including the very opposite (and very easily seen as opposite) of the truth.3069

jdhuey said...

Ilíon,

Sure, in thoise "technical senses" by which one misuses the concept we term 'information', and the term.

When I point out that I’m referring to information in the technical sense, I’m trying to keep me and the reader from misusing the concept – the technical sense of information is indeed different than the colloquial usage. For the purpose of this discussion, I think that information in the technical sense of the term is the correct one to use.


Ah! So, "personal incredulity" *is* dispositive ... when it needs to be?

No I was simply noting the limits of my experience, knowledge and imagination. All of the information that we actually deal with is encoded on something physically real. I realize that you would perhaps like to have information exist in some non-physically real form (a disembodied mind, say) but if that type of information actually exists then the burden is on you to show it.


In the famous phrase, you're consusing the map for the territory; you're making the category error of conflating therepresentation of information with that information.

I don’t think so. That is why I specified using the technical sense of the word ‘information’. I am explicitly trying to separate the concept of ‘information’ from the concept ‘meaning’.


You're also begging the question: you're starting with the (false) assumption that only physical entities are real.

No, I’m starting from the observation that the only entities that we know are real are physical entities. Again, if you want to assert the reality of non-physical entities, the burden is on you to make that case.

The rest of what you wrote strikes me at first glance to be a rather incoherent discussion with lots of misconceptions running through it. I may respond to those comments if I ever get to the point where I think I understand what you are trying to say and it somehow turns out to deserve to be addressed.

B. Prokop said...

"No, I’m starting from the observation that the only entities that we know are real are physical entities."

That statement, as a response to being told that you are begging the question, is itself a masterpiece of begging the question. I know that my consciousness is real. I do not know that it is a physical entity. You may claim that it is, but as you yourself wrote, "the burden is on you to make that case."

jdhuey said...

That statement, as a response to being told that you are begging the question, is itself a masterpiece of begging the question. I know that my consciousness is real. I do not know that it is a physical entity. You may claim that it is, but as you yourself wrote, "the burden is on you to make that case."

I don't think that I was 'begging the question' in either case. I was simply starting with what is known and should in and of itself be uncontroversial.

I don't know that consciousness is a physical entity and I certainly don't know that it isn't a physical entity. I do know that the current state of neuroscience investigations indicate that consciousness is a process that only takes place in a physical entity. And, it is also the case that if we assume that Quantum Field Theory is true (and there is no evidence that it isn't) then we know all the outside forces, fields and particles that could interact with our brains and none of those can account for consciousness. So, based on that I conclude that when it comes to our minds normal everyday reality is all that we have to work with.

B. Prokop said...

As for the observed reality of immaterial (?non-material?) entities, allow me to re-surface an argument I made on DI a long time back. To wit:

There is something which we all agree is the literary work The Brothers Karamazov. And I think we all agree that The Brothers Karamazov does exist in reality. But what is it? (Meaning: of what is it composed?) Is it ink on the paper it's printed on (or the magnetic or electrical patterns in a digital storage medium)? But if that's the case, then are there many Brothers Karamazovs? And if you burned every last copy, would it then cease to exist, despite there being many people who can remember reading it? Or does The Brothers Karamazov exist independently of the actual physical medium on which it is recorded?

I contend that there are not tens of thousands of Brothers Karamazovs out there, but only one, which we have access to via a multitude of varying physical means, and which is independent of these differing physical manifestations.

The Brothers Karamazov is a real thing which, is one thing (i.e., not many), and in and of itself, has no physical form, is made up of no material substance, and yet exists and is observable.

jdhuey said...

B. Prokop,

So, you hold with Platonic Forms. I don't (go figure.)

I would say that if all copies of the work were destroyed then no the work does not cease to exist while it exists in peoples memory. If the people who read the book were to recreate the book again then they can to the degree that their memories are accurate. If no one remembers the book then yep, it has ceased to exist.

B. Prokop said...

I'm not disputing that it could cease to exist, but while it does exist, of what is it composed? Certainly nothing material. (And if you answer "yes", then you have to say that there is not one Brothers Karamazov, but thousands (?millions?). Absurd!

So punt to Plato if you so desire - bottom line is that here we have an observable, immaterial object.

B. Prokop said...

Along the same lines, does Mahler's Second Symphony only exist while someone is listening to it, or does it exist all the time? If the former, then we have the bizarre idea of the symphony existing one moment and not the next, over and over and over again. And if that's indeed the case, is it the same symphony each time, or is it a completely new entity each time it's played?

If you discount the immaterial, you end up with all sorts of absurdities like that.

B. Prokop said...

Perhaps, jdhuey, one problem here is that you're not going to be satisfied that the case for the existence of non-material objects has been made until you're shown something that looks very much like a material object! But of course, by definition, the non-material is not going to exhibit the same properties as a material one, and why should it? Otherwise, you'd simply be demonstrating the existence of yet another material object.

So just maybe the reference to Plato's Ideals is apropos. The old guy was, after all, on to something there.

I just gave a few examples above, but for me personally (I have no expectations of convincing anyone else with the following argument, but I don't really care. It works for me.) the clincher is my own identity. Medical science has demonstrated that our bodies are continually rejuvenating themselves, cell by cell replacing itself. Over the course of 8-9 years, nothing is left of our former selves other than our teeth and a stray atom here and there. We are (physically) completely new beings. But I am unquestionably the same person that went under the name Bob Prokop 20 or 30 (or 62) years ago. This despite the fact that no physical remnant of that previous individual today exists. To me, this is proof beyond reasonable doubt that the entity known as "Bob Prokop" is not a physical object, or in any way dependent upon any physical structure, but is non-material (and will survive my body's demise).

World of Facts said...

Mr. Prokop said...

"I know that my consciousness is real. I do not know that it is a physical entity. You may claim that it is, but as you yourself wrote, "the burden is on you to make that case.""

There you go; the primacy of consciousness. This is the source of all these disagreement on what it means to be material or not, physical or not, natural or not.

"If you discount the immaterial, you end up with all sorts of absurdities like that."

If you start with the immaterial and then try to explain away the immaterial, of course you run into absurdities. But if you start with the material and can account for the immaterial, then materialism is more and more likely to be true. If no immaterial things stand on their own, materialism is as proven as it can be, which is the case.

World of Facts said...

Ilíon said...
" hugely erroneous: "[...]
God-haters have no shame; they will say *anything* including the very opposite (and very easily seen as opposite) of the truth
"
- Thanks for the nickname
- I don't hate God... you're weird.
- Any point to make? Any non-truth to point out or you're just rantin?

B. Prokop said...

"If no immaterial things stand on their own"

We, being denizens of this physical universe, should not expect to run across purely non-material entities except through their interaction with material ones, unless that entity chooses to make itself known through other means. Thus the possibility of Revelation is entirely reasonable, and poses no threat or contradiction to the physical sciences. As Fr. Robert Barron expressed it HERE, "faith is not infra-rational but rather supra-rational, that is to say, not below reason but above reason and inclusive of it."

Crude said...

I do know that the current state of neuroscience investigations indicate that consciousness is a process that only takes place in a physical entity.

Because neuroscience has investigated non-physical entities to determine whether or not consciousness takes place in such?

No, that's can't be right. Neuroscience is strictly limited to a certain kind of investigation, a certain kind of entity.

But at least neuroscience demonstrates that consciousness is a physical process, right?

Wait, no again, since consciousness - the subjective state itself - isn't accessible to experiment. Instead, neuroscience works with what are taken to be correlations, framed* as physical - a person moving, or making sounds - and comments on those. Useful stuff! But not what we're after here.

(* I say framed because science as science is metaphysically silent. Idealism could be true, for all science can tell, and for all that it matters, or any other things. Science's scope is limited, as are its models.)

And this:

The main problem with the AfR is that the proponent always start with the notion that minds can and do exist without a body,

Is nonsense. The proponent may believe that, but the argument simply states what must follow given pretty uncontroversial premises about what it is to reason, and what material processes are. It leads to conclusions people dislike, and so it's declared that it must be wrong.

Ilíon said...

B.Prokop "So punt to Plato if you so desire - bottom line is that here we have an observable, immaterial object."

To be more precise, The Brothers Karamazov is an example of an objectively (or publicly) observable immaterial object, whereas each person's mind is to himself a subjectively (or privately) observable immaterial object.

Which distinction helps highlight a small part of what’s wrong (on so many levels) with God-haters – they hate, and desire to eliminate, subjects (which follows from their hatred of The Subject).

Ilíon said...

B.Prokop "Perhaps, jdhuey, one problem here is that you're not going to be satisfied that the case for the existence of non-material objects has been made until you're shown something that looks very much like a material object!"

Exactly! “Until you can *reduce* immaterial entities to material entities, I will refuse to acknowledge that there are immaterial entities … and until I acknowledge them, you have failed to make the case for them.

B. Prokop said...

RE: What Ilion said.

And, since (whether they like it or not) the God-haters are nevertheless created in the Image of God, they inevitably get around to hating themselves. Thus the passionate desire to deny the existence of consciousness or free will.

Reminds me of the awesomely horrifying scene near the end of C.S. Lewis's novel That Hideous Strength, where the character Frost is strenuously using all his will power (which he denies that he possesses) to deny his own conscious existence, all the while being burned alive in a building fire which he himself started.

Ilíon said...

B.Prokop "I just gave a few examples above, but for me personally (I have no expectations of convincing anyone else with the following argument, but I don't really care. It works for me.) the clincher is my own identity. Medical science has demonstrated that our bodies are continually rejuvenating themselves, cell by cell replacing itself. Over the course of 8-9 years, nothing is left of our former selves other than our teeth and a stray atom here and there. We are (physically) completely new beings. But I am unquestionably the same person that went under the name Bob Prokop 20 or 30 (or 62) years ago. This despite the fact that no physical remnant of that previous individual today exists. To me, this is proof beyond reasonable doubt that the entity known as "Bob Prokop" is not a physical object, or in any way dependent upon any physical structure, but is non-material (and will survive my body's demise)."

The persistence of identity (*) happens to be a sub-argument of my “You are the proof of God” argument.

(*) I’m leaning toward the view that, at least of physical entities, only those things with identity actually exist. That is: human beings and animals (and perhaps plants, but certainly their cells) really do exist, but stones and the earth (and sun and moon) do not really exist.

B. Prokop said...

"I’m leaning toward the view that, at least of physical entities, only those things with identity actually exist. That is: human beings and animals (and perhaps plants, but certainly their cells) really do exist, but stones and the earth (and sun and moon) do not really exist."

Interesting. That is almost precisely what a physicist I spoke to last year, who set me straight about some misconceptions I had about quantum mechanics, told me. I asked Dr. Ron Lee (who unfortunately died just a few months ago) where was the information stored in fundamental subatomic particles that enabled those particles to "know" how to behave in various circumstances. (e.g., how does an electron know that it must avoid (i.e., be repelled by) another electron, if the electron is a unitary particle with no further substructure?). He patiently explained to me that I had it backwards. It wasn't that there was no place to store the needed information at that level, it was rather that information was all there was at the fundamental particle level. What we think of as "matter" (i.e., material objects) did not really exist when you came down to it. "Matter" was an informational construct of the sum total of all that was occurring at the particle level.

Papalinton said...

Just because the word 'god' exists doesn't substantiate the existence of the non-existent. The word 'god' has the same claim to actuality as does the word 'unicorn'.

George H Smith, author, educator, from the Cato institute and the Institute for Humane Studies (IHS) [out of George Mason University] eloquently characterizes it:

"God is not matter; neither is non-existence. God does not have limitations; neither does non-existence. God is not visible; neither is non-existence. God cannot be described; neither can non-existence."

The immaterial supervenes the material. Without the physical/material there can be no immaterial. That is simply a matter of logic, and uncontroversial.

B. Prokop said...

"Without the physical/material there can be no immaterial. That is simply a matter of logic, and uncontroversial."

How can it be uncontroversial when 99% of humanity would disagree with that statement? Ohhh... I see. It's uncontroversial because no one (well, hardly anyone) actually believes it.

I'm glad we got that one cleared up!

Oh, by the way. Since you say it is "simply a matter of logic", could you kindly show us the syllogism that led you to such a conclusion?

Ilíon said...

B.Prokop: "And, since (whether they like it or not) the God-haters are nevertheless created in the Image of God, they inevitably get around to hating themselves. Thus the passionate desire to deny the existence of consciousness or free will."

Yep, that's where my thought was going: in the end, they (hate and) deny the reality of human selves because they hate (and deny) the reality of the Self of Selves; the twisted logic of their hatred of and denial of God compels them ultimately to deny that they themselves are real.

The human self -- embodied, yet immaterial nonetheless, and *free* -- is the ultimate proof of the reality of the transcendent-of-nature Creator-God; getting rid of him requires getting rid of us. It’s similar to what we’ve already seen in just this thread, with the too-too common tactic of the God-haters, whereby they, who do beg the question constantly, falsely accuse any non-materialist of begging the question when he presents a reductio ad absurdum of materialism (which, among other things, the AfR is); their twisted reasoning going something like this: “This hateful-to-me conclusion is the one he was aiming for, so he *must have* cheated to get it!” And since begging the question may be the most common error of reasoning that humans make (and certainly the one with which they are most intimately familiar), that’s what they accuse. In both cases, the twisted “conclusion” (“God is not”; “you didn’t logically show that determinism (*) does not fully explain reality”) comes first, and everything else they say follows from the need to protect that “conclusion”.


(*) materialism or physicalism or naturalism, whatever one wants to call it, is really only a species of determinism.

Ilíon said...

"Interesting. That is almost precisely what a physicist I spoke to last year, who set me straight about some misconceptions I had about quantum mechanics, told me."

In my case, I started considering that after contemplating the "paradox" of identity, à la Perseus' Maserati ;)

Ilíon said...

"Ohhh... I see. It's uncontroversial because no one (well, hardly anyone) actually believes it."

I haven't seen *any* evidence that anyone at all believes that "Without the physical/material there can be no immaterial" -- the continuous behavior of those who assert that demonstrates that they don't really believe it.

amorbis said...

I've said it before and I'll say it again - I don't think the existence of an immaterial soul proves, beyond reasonable doubt, the existence of God. There are always coherent ways of describing reality that make reference to a soul without making reference to God, Buddhism being a prime example. To be sure, the existence of a soul, if proven, would provide considerable inductive evidence for God, considering that the "hypothesis" of God (under most formulations) predicts that souls will exist. But I don't think a strict deductive argument from the existence of a soul to the existence of God can be made to work (at least not in a way that doesn't simply collapse into a contingency argument). If you want to argue for the existence of God, you're better off looking elsewhere for a deductive proof.

B. Prokop said...

In the spirit of I've said it before and I'll say it again, I believe that attempting to "prove" the existence of God is starting off on the wrong foot. Very few people have come to the Faith through reason alone. That in no way implies that they did so contrary to reason, but simply means that there were other factors, equally valid and of perhaps greater importance, that came into the equation. After all, we must never forget the admonition of Saint James: "You believe that God is one; you do well. Even the demons believe - and shudder." An intellectual assent to the truths of the Faith may be essential (as is the foundation of a house), but just as one cannot live in a foundation, simple belief in God (by itself) gets one nowhere.

Two more points, one rather fuzzy, and the other a bit more clear.

1. Faith is listed amongst the Theological Virtues for a reason. There's more than mere intellect at stake here.I quite firmly believe that, in the end of ends, a lack of Faith is unvertuous. Atheists, such as Boghossian, accuse believers of willfully believing. To a certain (limited) extent, he is correct, in that unbelievers also willfully disbelieve. It is impossible to either believe or not to believe absent an act of will.

(That was the "fuzzy" one.)

2. Christ Himself refused to make belief mandatory. In the Wilderness, the devil offered three temptations to our Lord, each of which involved a perversion of His mission on Earth. 1) "Command these stones to become loaves of bread." I.e., remove pain and suffering from your creation, and people will be compelled to believe. 2) "Throw yourself down [from the Pinnacle of the Temple]." I.e., Compel belief through "signs and wonders" (as are so often demanded by atheists). And 3) The devil took [Jesus] to a very high mountain, and showed him all the kingdoms of the world and the glory of them; and he said to him, "All these I will give you, if you will fall down and worship me." I.e., Simply remove Free Will and people will have no choice but to believe.

Christ rightfully rejected these Satanic suggestions. And the fact that atheists demand the very same to this day speaks volumes.

Papalinton said...

Out of interest, who are the god haters on this blog, Illion?

I am a humanist
Are humanists god haters?.

I am a naturalist.
Are naturalists god haters?

I am a good person.
Are good people god haters?

I love humanity and am a humanitarian, a person who seeks to promote human welfare.
Are humanitarians god haters?

What is a god hater? What does it look like?







Papalinton said...

Bob, I read your Point 2 about Satan and Jesus in the desert. I then read this from another religion:

"The thetan is the true identity of a person – an intrinsically good, omniscient, non-material core capable of unlimited creativity.[98][99] In the primordial past, thetans brought the material universe into being largely for their own pleasure.[98] The universe has no independent reality, but derives its apparent reality from the fact that most thetans agree it exists.[99] Thetans fell from grace when they began to identify with their creation, rather than their original state of spiritual purity.[98] Eventually they lost their memory of their true nature, along with the associated spiritual and creative powers. As a result, thetans came to think of themselves as nothing but embodied beings.[99][101] Thetans are reborn time and time again in new bodies through a process called "assumption" which is analogous to reincarnation.[98] Like Hinduism, Scientology posits a causal relationship between the experiences of earlier incarnations and one's present life, and with each rebirth, the effects of the MEST universe (MEST here stands for matter, energy, space, and time) on the thetan become stronger."

Both are very strange narratives. And yet both are as fervently believed in as the other. What makes Thetan belief any more substantive than Christian belief? One wonders?

B. Prokop said...

Your second example (and yes, I know where it comes from) has no bearing on the subject and no place in this discussion. (And yes, you also know that very well.)

Is that all you got? 'Cause if'n so, then it's game, set, and match. You're simply proving my Point Number One - it takes an act of will to disbelieve.

"Nothin' to see here, folks... Move along!"

B. Prokop said...

Linton,

Still waiting for that syllogism, by the way.

World of Facts said...

Mr. Prokop said:
"should not expect to run across purely non-material entities except through their interaction with material ones, unless that entity chooses to make itself known through other means"

How can a purely non-material entity interact with a material one? And would a purely non-material entity be, or explained? All non-material entity I hear people talk about are accountable by the existence of the material, and the claims of non-material existence outside of the material world are always assumed, not justified.

" the possibility of Revelation is entirely reasonable, and poses no threat or contradiction to the physical sciences."

A Revelation would still have a physical component to it. The physical human receiving the revelation is presumed to be alive and conscious. The way I understand a revelation, it's nothing more than a human thinking about something new.

"As Fr. Robert Barron expressed "

Funny, I interacted with him on his YouTube channels years ago; one of the first online conversation I ever had. Interestingly enough, the notion of what a non-material thing is came up, and he gave me 1 example and 1 example alone: an angel. As if angels were supposed to be something obviously real.

"As Fr. Robert Barron expressed it HERE, "faith is not infra-rational but rather supra-rational, that is to say, not below reason but above reason and inclusive of it.""

Yes I am well aware of the gymnastic used by non-rational believers to conclude they are rational. A good friend of a friend once told me something very similar. He was a member of the Opus Dei and used that very same expression of 'supra-rational', or I think 'super-rational' were the ones he used, not much difference I guess. It's simply a way to justify the un-justifiable. If something cannot be supported by reason, just make up your own super-reason and voila!

World of Facts said...

Crude said...
"The main problem with the AfR is that the proponent always start with the notion that minds can and do exist without a body,

Is nonsense. The proponent may believe that, but the argument simply states what must follow given pretty uncontroversial premises about what it is to reason, and what material processes are.
"

Exaclty, the proponent believes that and cannot even consider it to be wrong. The primacy of consciousness is a base point, which is never questioned, and leads to the acceptance of wrong arguments such as the AfR. Show me your favorite version and I will point out to you where it fails because of this assumption.

" It leads to conclusions people dislike, and so it's declared that it must be wrong."

Nice try at psychoanalyzing. Again. A failure. There is nothing to like or dislike about the conclusion. It's just wrong.

Ilíon and Mr. Prokop said...
"what’s wrong (on so many levels) with God-haters – they hate, and desire to eliminate, subjects (which follows from their hatred of The Subject).
[...]
God-haters are nevertheless created in the Image of God, they inevitably get around to hating themselves.
"

I also don't get where this 'hate' comes from or where you see it.

"Exactly! “Until you can *reduce* immaterial entities to material entities, I will refuse to acknowledge that there are immaterial entities … and until I acknowledge them, you have failed to make the case for them.”"

Your caricature fails to capture the essence of the position. The point is that all the immaterial entities you talk about can be accounted for by the material world. The concepts you express are all "in your head", in your consciousness, and depend on the existence of your own physical self to exist, or others. The examples of purely non-material entities are only defined as purely non-material but never justified.

In other words, we have a bunch of things we can conceptualize, and some of them point to material things while others point to non-material things, abstract constructs or concepts, but they in turn end up pointing to material things and ultimately fit into a purely material world. So it's not that we refuse to acknowledge the immaterial entities; there is just no reason to believe they exist, other than as human constructs.

"Thus the passionate desire to deny the existence of consciousness or free will."

2 things that physical human beings possess. They are easily defined in physical terms and represent characteristics of physical human beings. No problem unless you start with the primacy of consciousness, which you do, and then fail to see how it's even possible for your mind to be physical. Of course, you define it as non-physical to start with.

B. Prokop said...

"non-rational believers"

Interesting adjective. So you think that Georges Lemaitre, St. Thomas Aquinas, Gregor Mendel, Augustine, Maxmilian Kolbe, Copernicus, Dante, Andre Rublev, Johannes Kepler, C.S. Lewis, Dorothy Day, Sir Isaac Newton, Saint Ignatius of Antioch, Athanasius of Alexandra, Aleksandr Solzhenitsyn, and Johann Sebastian Bach are all "non-rational"?

Like I said, interesting.

World of Facts said...

No, let me correct this right away; it's not that the person is non-rational; the belief in these specifics arguments is. And yes, even smartest of us can believe irrational things. Newton is always the best example.

B. Prokop said...

"even smartest of us can believe irrational things"

Atheists prove the validity of this every day.

im-skeptical said...

Ilion's reductio ad absurdum:

Believing that mind can be purely material is just stupid, because anyone who's not a god-hater knows that it can't be. Therefore mind must be immaterial. And how dare you falsely accuse me of begging the question.


World of Facts said...

"even smartest of us can believe irrational things"

"Atheists prove the validity of this every day."

Ok but you need to add, 'believe irrational things' or 'reject rational beliefs' then
;-)

World of Facts said...
This comment has been removed by the author.
grodrigues said...

@Hugo:

"Show me your favorite version and I will point out to you where it fails because of this assumption."

The owner of this blog wrote an entire book on the subject; then contributed a whole article to The Blackwell Companion to Natural Theology. I haver the read the latter (and can get my hands on a copy) but not the former.

So let's start here. Tell us where the argument goes wrong with quotes and page number references.

World of Facts said...

Seems accurate im-skeptical, but who knows, maybe Ilíon is not really a human being with a physical body. How can we prove that he's not an immaterial mind interacting with us, mere fallible physical beings, through the internet via some immaterial-to-material device?

B. Prokop said...

"Believing that mind can be purely material is just stupid"

Which is amply demonstrated by the fact that Skep so believes. It's amazing the things Skep can believe in (or not believe in). He believes his consciousness is purely material, yet acts as though it is not. He denies free will, yet behaves as though it existed. He is even capable of believing with no doubt things he has no evidence for, whilst simultaneously claiming "the evidence is out there!" (Funny thing about that evidence. He never seems to be able to produce any.)

World of Facts said...

grodrigues, if you give me a book I'll give you a book:
http://infidels.org/library/modern/richard_carrier/reppert.html
Victor already replied to that critique on his blog too; you might have seen that already.

For 1 quick example of the primacy of consciousness' bias, look at this point Carrier quoted as the 'most faulty, from Victor's book:
"No belief is rationally inferred if it can be fully explained in terms of nonrational causes."
This premise is accepted because of the notion that minds are assumed to be non-material. It's indirectly implied by contrasting 'rationally inferred' with 'nonrational causes'.
- When stating that something is 'rationally inferred', the author implies the use of reason, something minds do. Because minds are defined as non-material and capable of reasoning, a belief is labeled as 'rational' if it came from a non-material mind
- 'nonrational causes' on the other hand refers to the notion that material things are assumed to be nonrational, incapable of reasoning. This follows from the assumption that minds, who do reason, are immaterial.
Therefore, the author is saying that if a belief is rationally inferred, it is necessarily caused by a mind, and if it is caused by a mind, it is thus caused by a non-material entity. If beliefs were explained by material causes, they would not be rational, because they would not be the product of a rational non-material mind.

Non-material rational minds are assumed to exist first, for no good reason.

B. Prokop said...

Hugo,

I had never come across the term "primacy of consciousness" before you brought it up earlier in this conversation. Had no idea what it meant, so I searched around the web for some explanation. Now that I've done so, I have to confess that I still have no confidence whatsoever that I understand what it's supposed to be. That said, I must respectfully dissent from your characterization of what I posted yesterday as being an example of "primacy of consciousness", since that would mean subscribing to an idea of uncertain significance or import. I can neither agree nor disagree with it until I figure out what it actually means.

jdhuey said...

Hugo,

"- 'nonrational causes' on the other hand refers to the notion that material things are assumed to be nonrational, incapable of reasoning."

So, do I have this right: a piece of metal can not do reasoning because it is a non-rational material thing.

What about a calculator that is just that piece of metal reconfigured. The metal is still just as non-rational as before but now when a physical force (also non-rational) pushes the metal it performs an arithmetic calculation (a rational process).


grodrigues said...

@Hugo:

"No belief is rationally inferred if it can be fully explained in terms of nonrational causes.

This premise is accepted because of the notion that minds are assumed to be non-material. It's indirectly implied by contrasting 'rationally inferred' with 'nonrational causes'."

From the man himself:

"The second fallacy Carrier says I commit is the Causation Fallacy. He thinks that I, along with C. S. Lewis, endorse the argument that “the presence of a cause and effect account of belief is often used to show the absence or irrelevance of a ground and consequent relationship,” and that therefore all cause and effect accounts prove the absence or irrelevance of ground and consequent relationships. However, he claims in arguing thus I commit the fallacy of Affirming the Consequent, Hasty Generalization and Red Herring. (Quite a lot of fallacies to commit in one argument!) But while this type of argument was used in Lewis’s original chapter, in the revised edition this is not the primary reason for thinking that reason cannot be accounted for naturalistically. And I don’t argue that way either. The argument against the relevance of ground and consequent relationships has to do with the causal closure of the physical. If a physical account of the process is causally complete, and physics is mechanistic, how do reasons come into play? Remember, at the most basic level of analysis physics, in order to play the role of physics in the kind of physicalism that is under attack in my book, must be mechanistic. This being the case, if we apply the Principle of Explanatory Exclusion and the Principle of Causal Exclusion defended by Kim and others, a case can be made that a comprehensive physicalistic causal explanation excludes a mentalistic account of rational inference."

IOW, you have no idea what you are talking about.

grodrigues said...

@jdhuey:

"The metal is still just as non-rational as before but now when a physical force (also non-rational) pushes the metal it performs an arithmetic calculation (a rational process)."

The metal does not perform anything, much less "an arithmetic calculation". What a physical description of what is going on tells us is that some electrons shift this and that way (or whatever); this is then *interpreted* by us, rational thinkers, as performing an arithmetical calculation (as opposed to decompressing a zip file, executing a code fragment in some exotic assembler language or whatever) and hinges on all sorts of conventions and derivative intentionality of rational thinkers. But at the level of the physical description, which is what Reppert is targeting, there is no "arithmetical calculation" going on.

So to show that pieces of inorganic matter think or can think, you pick as an example precisely an *artifact* constructed by *human beings*. The irony is sweet.

note: this also fails to get the nature of rationality right, but since this is harder to explain I will leave it be.

jdhuey said...

@grodriques,

"So to show that pieces of inorganic matter think or can think, you pick as an example precisely an *artifact* constructed by *human beings*. The irony is sweet."

1. I didn't say the piece of metal could think. I said that it performed an arithmetic calculation. I have always thought that arithmetic calculations were an example of a rational process. You seem to think I'm mistaken.

2. I don't think that the origins of the calculating device is particular relevant. I see our brains as the product of natural selection and, as such, the calculator is just an extended phenotype of our genetic legacy.

jdhuey said...

@grodriques,

"...this is then *interpreted* by us, rational thinkers, as performing an arithmetical calculation..."

As I see it, when we do this *interpretation* our material brains are 'shifting electrons about' and that is performing another rational process that differs from the arithmetical calculator only in that it is vastly more complex.

So, I think that you have the burden to show that the *interpretation* can not be a rational process (as with the calculator)carried out by a material object (our brains). Otherwise, you have done exactly like Hugo has stated and simply begged the question.

amorbis said...

jdhuey,

You've just opened up a vicious regress. The calculator's operation doesn't even *count* as "calculation" insofar as we interpret it as such, because intrinsically all it is is "shifting electrons about". If our interpretation of it as calculating is *itself* merely "shifting electrons about", then our *act of interpretation* only counts as interpretation insofar as something else interprets it as such. Bam, you've got an infinite regress of interpretation.

B. Prokop said...

"I don't think that the origins of the calculating device is particular[ly] relevant."

It is extremely relevant (just perhaps not to the current conversation). It is very strong evidence in favor of ID, as has been explained in extensive detail in previous conversations on this website.

jdhuey said...

@amorbis,

Yep, the self-referential feedback over different physical and computational levels can create what Douglas Hofstadter called a strange loop.

http://en.wikipedia.org/wiki/I_Am_a_Strange_Loop

jdhuey said...

@Bob,

"It is extremely relevant (just perhaps not to the current conversation)."

That's okay, a conversation for another day.

(BTW, my name is John Huey, so if you would prefer you can call me John instead of the jdhuey screen name.)

amorbis said...

It's amazing the kind of incoherent BS materialists will defend in their desperation to defend the dogma that the physical is all there is. It's sad, really.

amorbis said...

The vicious regress is a well-known problem with ascriptionist theories of intentionality (like Daniel Dennett's), and there's a reason such theories are by and large rejected even by materialists.

jdhuey said...


@amorbis,

While I don't necessarily buy Hofstadter's Godellian strange loop hypothesis as the foundation of consciousness, I certainly would not call it either incoherent nor BS.

That the physical is all that there is* is not "dogma". It is simply the best conclusion that we have given what we know of reality.

(*I dislike using the term 'physicalism' to describe 'naturalism' because naturalistic reality also contains fields and forces and virtual particles and information and entropy and order and complexity and so on. So, yes, there is more than just 'the atoms and the void' but we have no reason to think that the 'more' contains anything supernatural.)

William said...
This comment has been removed by the author.
B. Prokop said...

"That the physical is all that there is ... is simply the best conclusion that we have given what we know of reality."

Hmm... Best for you, maybe. Have you ever considered that your conclusion rests on your (apparently unexamined) priors? I presumably know the much same things about reality as you do, yet I think the idea that the physical is all there is is a terrible conclusion - fatally flawed, logically incoherent, and fundamentally contradictory.

B. Prokop said...

Arrgh! You deleted the comment I was responding to!

amorbis said...

That the physical is all that there is* is not "dogma". It is simply the best conclusion that we have given what we know of reality.

It is most certainly *not* the best conclusion we have given what we know of reality. Maybe if you ignore consciousness, intentionality, mathematics, (at least some) universals, etc. it becomes the best conclusion we have, but for those of us who want to include all aspects of reality in our worldviews (and not just what science can observe and measure), materialism is simply a non-starter.

William said...

The disagreement here is between those who think "the physical" encompasses mental and abstract entities, and those who do not.

If we require the physical to only include mention the properties included in quantum mechanics, we cannot even state the laws of quantum mechanics, since they are abstract.

If we define the mental as physical, then all we have left is a causal chasm between the microphysical properties (which do not determine much else of the putatively physical) and everything else.

jdhuey said...

"The vicious regress is a well-known problem..."

Infinities, Infinite regress and Infinitesimals have been a problem for people that limit there math and logic to that of the ancient Greeks (i.e. Zeno, Aristotle, etc.) The Jesuits certainly had a problem with those concepts - they went after Galileo as much for his use of infinitesimals* as for promoting heliocentrism. But for anyone that has accepted the modern mathematics of Calculus and Cantor's method of dealing with infinities they are not a problem.

(Galileo didn't exactly use infinitesimals as was later developed by Newton, Libniz, and others but rather he used a precursor concept called Indivisables. Indivisables still relied on the concept infinitely small distances though.)

Papalinton said...

Yes Bob. I read your demand for a syllogism. Unfortunately, the use of syllogisms in the contemporary world is problematic at the best of times as they in large part reflect an adherence to an archaic paradigm of philosophical thinking that has not kept up with modern thinking. This is a very interesting summation of today's perspective around the use of syllogisms:

"The Aristotelian syllogism dominated Western philosophical thought for many centuries. In the 17th century, Sir Francis Bacon rejected the idea of syllogism as being the best way to draw conclusions in nature.[10] Instead, Bacon proposed a more inductive approach to the observation of nature, which involves experimentation and leads to discovering and building on axioms to create a more general conclusion.[11] ............... This led to the rapid development of sentential logic and first-order predicate logic, subsuming syllogistic reasoning, which was, therefore, after 2000 years, suddenly considered obsolete by many.[original research?] The Aristotelian system is explicated in modern fora of academia primarily in introductory material and historical study. One notable exception to this modern relegation is the continued application of Aristotelian logic by officials of the Congregation for the Doctrine of the Faith, and the Apostolic Tribunal of the Roman Rota, which still requires that arguments crafted by Advocates be presented in syllogistic format." [Wiki]

So you can understand my reticence in offering a syllogism.

im-skeptical said...

Amorbis' infinite regress is a direct result of his dualism. There has to be a little man inside the material brain that serves as the interpreter. But how does this homunculus understand? Well, it's the little man inside him ...

This is the absurdity of dualistic thinking - your a priori belief that understanding can ONLY be done by an immaterial little man inside us, which would end the regress. Dump your dualism. It's incoherent.

jdhuey said...

"Have you ever considered that your conclusion rests on your (apparently unexamined) priors? I presumably know the much same things about reality as you do, yet I think the idea that the physical is all there is is a terrible conclusion - fatally flawed, logically incoherent, and fundamentally contradictory."

Well, I'm certainly open to the idea that I might have some priors that need to be examined but I would have to have them pointed out to me because I like to think that I evaluate my own thinking with as much rigor as I evaluate others. I don't think that I'm wrong (obviously) but if I am I want to know about it.

B. Prokop said...

"So you can understand my reticence in offering a syllogism. "

No, I cannot. It was you yourself that introduced the word "logic" into the discussion. You wrote "Without the physical/material there can be no immaterial. That is simply a matter of logic". Well then, if that is the case, you ought to be able to present a logical argument. Otherwise, it is not simply a matter of logic.

B. Prokop said...

" Dump your dualism. It's incoherent."

It's far less than the incoherency of your bleating that "the evidence is out there", yet failing to provide the least shred of such. Let's hear it, Skep. Right here, right now:

- Identify the portion of scripture that has been modified to comply with Church dogma (chapter and verse, please).

- Give us the "before" version, so we can confirm that it has actually been modified.

- Explain what dogma said modification is being complied with.


Pull the scab, Skep. One good tug, and it's over. Cough up the evidence.

im-skeptical said...

Just read what I gave you, Bob.

B. Prokop said...
This comment has been removed by the author.
B. Prokop said...
This comment has been removed by the author.
B. Prokop said...

I will quote myself. I just posted this under the heading "Leaving the fold, atheist style" (five conversations below this one):

I DID click on your links, including the one to the Islamic extremist website (nice touch there - always give 'em the unexpected). They were not quite up to your classic "Whore of Babylon" conspiracy websites, but hey, you can't expect to bowl 300 every day. And as to not paying attention, heck, I gave you a line by line critique of one of them. Ooohh... I see, if I don't immediately come to the same batshit conclusion you do, then I'm "not paying attention".

Not good enough, Skep. Give it to us here, on DI. Otherwise, ya got nothin'! I read your links, and there was NO relevant evidence in them. If you dispute that, then it ought to be a piece of cake for you to copy/paste it onto a posting here and prove me wrong. If you can't do that, then you are either:

a) a liar, or
b) a fool

There is no third alternative.

William said...

Skep,
you persist in attacking a strawman version of dualism. All dualists are doing is defining a subset of what you think to be physical as non-physical.

And as far as homunculi go, there are lots of them in the brain, it's only the philosophers who have issues with them :).

Papalinton said...

Sorry Bob, but 'logic' entails a deeper and wider commission than the simpleton understanding of syllogisms. The citation I provided makes it very clear. All fact or truth cannot be contained within a syllogism. Now that observational and testable scientific methodology has reached full bloom adherence to a pre-scientific syllogistic paradigm is a partial explanation at best and an epistemologically poor one at that. It worked well at a time when our systematic understanding of nature was woefully inadequate.

We have moved on to far more advanced, sophisticated and nuanced explanatory means that outstrips syllogistic logic. Perhaps you haven't noticed the change yet as a result of your perfervid pre-occupation with old-time theological explanations. But it is occurring nonetheless.

B. Prokop said...

Well, Linton, if you can't give one, then just say so... No need for all the 3 dollar words.

Papalinton said...

Skep
There is no doubt that christian believers subscribe to the homunculus conception. They either simply refuse to admit it or are incapable of discerning it. The entity that resides in the brain of every believer, is none other than god, the great christian homunculus in person. In fact there are two homunculi; Satan under whom all bad thoughts and deeds are agglomerated, and god, under whom all the good things they think and do are posited. Religious brains overflow with the dualistic battle between their Satanic impulses and their godly impulses, the dualistic tug-of-war with themselves, pulling them apart.

The pathology of religious belief indeed signals it as deeply incoherent.

grodrigues said...

@jdhuey:

"I didn't say the piece of metal could think. I said that it performed an arithmetic calculation. I have always thought that arithmetic calculations were an example of a rational process."

So a "rational process" is not an example of "thinking"? Ok.

"You seem to think I'm mistaken."

Yes, quite obviously I think you are mistaken. But since this sentence comes after an irrelevant terminological quibbling, it could be the case that you are mistaken about what I think you are mistaken about. But I admit I could be mistaken about this.

"I don't think that the origins of the calculating device is particular relevant."

Well, so the irony is lost on you.

"So, I think that you have the burden to show that the *interpretation* can not be a rational process (as with the calculator)carried out by a material object (our brains). Otherwise, you have done exactly like Hugo has stated and simply begged the question."

Very funny.

"Infinities, Infinite regress and Infinitesimals have been a problem for people that limit there math and logic to that of the ancient Greeks (i.e. Zeno, Aristotle, etc.)"

Infinite regress is a conceptual problem that is not dissolved by appealing to, or or even has any remotely relevant link with, mathematical infinities or infinitesimals.

"The Jesuits certainly had a problem with those concepts - they went after Galileo as much for his use of infinitesimals* as for promoting heliocentrism."

No comments.

im-skeptical said...

William,

I speak of homunculus in the sense of Terrence W. Deacon's book:

http://www.amazon.com/Incomplete-Nature-Mind-Emerged-Matter/dp/0393049914

I am well aware that dualists recoil at the idea of their soul as a homunculus. But this is no strawman. This is precisely why Amorbis thinks there must be a regress.

im-skeptical said...

Bob,

Respond to the argument, not the fact that it is presented by a Muslim. (And I found no hint of terroristic tendencies there. It discusses many religions.)

William said...

skep,

Deacon seems to think that any teleological explanation is like a homunculus. But ridiculing teleology in biology with an inaccurate label does not eliminate it in any way. It's just a distraction from admitting the problem exists.

His definition also fails to do justice to the actual homunculi in the brain and what they tell us about its spatial organization.

amorbis said...

Skep,

You really need to think things through before you say them. Nowhere in what I said did I assume the truth of dualism. What I took as my starting point was the observation that a calculator's inputs, outputs, and internal processes only count as "calculating" insofar as we interpret them as calculating. Intrinsically, all that's going on is electrons moving about and photons coming out of the screen; it is us who give these intrinsically meaningless physical events meaning. But if physical processes are only meaningful insofar as they are interpreted as meaningful, and our interpreting is itself a physical process in the brain, then it follows that our brain processes only count as interpreting insofar as they are interpreted as interpreting. But then *that* interpreting only counts as interpreting insofar as it is interpreted as interpreting, and so on and so forth. That is where the infinite regress comes from - it comes from the thesis that physical processes are only meaningful insofar as they are interpreted as meaningful (a thesis that materialist reductionist hero Daniel Dennett would wholeheartedly agree with). Dualism is not assumed at any point in this argument.

B. Prokop said...

"Respond to the argument, not the fact that it is presented by a Muslim."

I'm not asking a Muslim for evidence of a slanderous accusation - I am asking you. You're the one who made it. Produce the evidence, or retract the accusation.

It's screamingly obvious that you cannot produce, so why do you persist in your calumny? If you have an example of a modified text in scripture, then identify it! And show us what it looked like before, so we can see that it was indeed modified. If you cannot do that, then you are guilty of making a baseless accusation without evidence, and are bereft of all honor and decency.

Oh, but honor and decency are "immaterial" concepts, and therefore do not have any real existence. I see... So that's how it works!

im-skeptical said...

Amorbis,

"it follows that our brain processes only count as interpreting insofar as they are interpreted as interpreting ..."

If that isn't dualist thinking, then there is no dualism. You should try to learn about how the mind works from a more scientific perspective.

World of Facts said...

Mr. Prokop said...
"I had never come across the term "primacy of consciousness" before you brought it up earlier in this conversation. Had no idea what it meant, so I searched around the web for some explanation. Now that I've done so, I have to confess that I still have no confidence whatsoever that I understand what it's supposed to be. That said, I must respectfully dissent from your characterization of what I posted yesterday as being an example of "primacy of consciousness", since that would mean subscribing to an idea of uncertain significance or import. I can neither agree nor disagree with it until I figure out what it actually means."

Makes sense. You can actually figure out where you stand by asking yourself what it means to exist and what you use as a basis for existence. Do you think that your mind, your consciousness exists regardless of what body you have, now and in the future, or do you think that you exist and are conscious because your material body exists? What you posted yesterday seems to point to the former:
"I know that my consciousness is real. I do not know that it is a physical entity."

I stand on the other side. I base my ontology on the real objective material world; it is a base reality. In this reality, things exist or not, they cannot exist and not exist at the same time. They either exist or they don't. Because my body exists in this reality, I can perceive it, interact with it and imagine it in my head, in what we refer to as a conscious experience. There is no doubt that the 'I' in my head is not an illusion because of this reality that my body lives in. I don't "think therefore I am", I am, I exist, and therefore I can think, talk, observe, reason and even argue with real physical people on the internet, who in turn think that they are non-material souls inhabiting a physical body.

World of Facts said...

grodrigues said...
"From the man himself:
[...copy/paste...]
IOW, you have no idea what you are talking about.
"

So you copy/paste something, insult me and move on. Not very impressive. Moreover, what you presented support my understanding of Victor's position; it does depend on the primacy of consciousness. I will address it quickly anyway, even if you did not bother trying to...

" If a physical account of the process is causally complete, and physics is mechanistic, how do reasons come into play? Remember, at the most basic level of analysis physics, in order to play the role of physics in the kind of physicalism that is under attack in my book, must be mechanistic. This being the case, if we apply [2 principles supporting Victor's position] defended by Kim and others, a case can be made that a comprehensive physicalistic causal explanation excludes a mentalistic account of rational inference."

In the first sentence, he asks how reason can come into play if all we have are physical processes. Then, he insists that a purely mechanistic world is what his book debunks. Then, another sentence is added to express how 2 principles defended by philosophers reject the idea that physical things alone can lead to rationality. Therefore, we are led to believe that because rational inference are mentalistic, non-material, we cannot possibly have a purely physical explanation for them, for if we had such explanation, they would not be rational inferences anymore... because they would be purely physical. Mental processes are defined as non-material first, and then "proven" to be not-only-physical because it would contradict that assumption.

But if we start with the physical world as a basis, assume that we exist in it as conscious physical beings; then it is possible that this conscious experience is nothing more than the result of the physical body. We are alive, can reason and have free will; we cannot deny that, just like we cannot deny the reality we live in. Hence, with these starting points, we can investigate what it means to think, to reason, to observe, to count, to feel and so on. This is not a proof that everything is physical; it's a starting point that provides a solid framework for what is real, what exists, and how we can go about to figure out what else exists beside these immediate things we know exist. It is anything but dogma that the physical is all there is.

There is still some difficult leap between the physical and the mental (we cannot explain how the electrons in the brain need to be arranged to make us experience love, just to name 1 example) but we don't solve that problem by simply defining conscious experiences as non-material and thus concluding that the material cannot explain them.

Mr. Prokop said...
"Have you ever considered that your conclusion rests on your (apparently unexamined) priors? I presumably know the much same things about reality as you do, yet I think the idea that the physical is all there is is a terrible conclusion - fatally flawed, logically incoherent, and fundamentally contradictory."

You were not addressing my claims yet when writing this, but I did express my priors up here and do think that the conclusion that the physical is all there is is likely to be true, though never quite possible to prove. Which part of my worldview is thus fatally flawed, logically incoherent, and fundamentally contradictory?

World of Facts said...

amorbis said...
" It is most certainly *not* the best conclusion we have given what we know of reality. Maybe if you ignore consciousness, intentionality, mathematics, (at least some) universals, etc. it becomes the best conclusion we have, but for those of us who want to include all aspects of reality in our worldviews (and not just what science can observe and measure), materialism is simply a non-starter. "

- Consciousness is a human experience; humans with physical bodies
- Intentionality is a human experience; humans with physical bodies
- Mathematics describe the real world we live in; tools created by humans who observed the world
- What universals can you express without referring to the physical world?
- What science observe and measure is not everything; abstract concepts and subjective experience exist, but are humans-dependant are not purely non-material either
- Materialism is a non-starter, correct, it's a conclusion that follows the rejection of supernatural claims

" if physical processes are only meaningful insofar as they are interpreted as meaningful, and our interpreting is itself a physical process in the brain, then it follows that our brain processes only count as interpreting insofar as they are interpreted as interpreting. But then *that* interpreting only counts as interpreting insofar as it is interpreted as interpreting, and so on and so forth. That is where the infinite regress comes from - it comes from the thesis that physical processes are only meaningful insofar as they are interpreted as meaningful"

'Meaningful' comes from the fact that humans ascribe meaning to things, yes, but this action, this experience, happens when a human look around using his/her physical bodies and compare the observation to the mental states representing prior observations, along with collaboration with others. There is thus no problem since the material comes first and allow humans to ascribe meaning to things. There is an infinite regress only if you start with your consciousness existing as a non-material thing and then try to make it meaningful in terms of physical terms, after the fact. So I agree completely with you when you say" it is us who give these intrinsically meaningless physical events meaning. " We give meaning to our own physical experience because we are living it, experiencing it.

Lost of claims and assertions, I know... please point out flaws and I will gladly correct!

William said...

Hugo,

Like skep, who claim that anything whatever that exists is physical, you claim that all aspects of human experience are physical, because bodies are physical.

This is a proof by definition, like skep's.

Let's take what you say to be true, but now separate the physcial aspects of humans that include things like meaning, thought, feeling from the rest of what is physical: let's call them "sphysical" instead.

Have we changed anything in your view? Not necessarily, just redefined part of the physical.

Now let us further say the the "sphysical" is NOT physical, by OUR definition.

That's dualism lite.

Your position, on the other hand, is kind of a physicalism lite, due to lack of commitment to reduction as something you know we can do Real Soon Now.

Very little practical difference.

grodrigues said...

@Hugo:

So let us take the first sentences of your quick address.

"In the first sentence, he asks how reason can come into play if all we have are physical processes. Then, he insists that a purely mechanistic world is what his book debunks. Then, another sentence is added to express how 2 principles defended by philosophers reject the idea that physical things alone can lead to rationality."

Kim mentioned in the quoted provided is Jaegwon Kim. So let's see in what those two principles consist, shall we?

(A) Principle of Explanatory Exclusion: there can be no more than a single *complete* and *independent* explanation of any one event.

(B) Principle of Causal Exclusion: If an event e causes event e*, then there is no event e# such that e# is non-supervenient on e and e# causes e*.

So these two principles can only be defended by philosophers with prior commitments antithetical to physicalism? Really? Jaegwon Kim used (B) to give a version of the causal closure argument for... physicalism, that is, for the thesis that mental events are supervenient on physical ones.

Let us continue with your scholarly and erudite display of knowledge.

"Therefore, we are led to believe that because rational inference are mentalistic, non-material, we cannot possibly have a purely physical explanation for them, for if we had such explanation, they would not be rational inferences anymore... because they would be purely physical."

All versions of the AfR are a deductive *argument* that purport to show that rational inference cannot be wholly reduced to the physical facts. Here you show a complete ignorance of what those arguments *actually* are and conclude, based on your ignorance, that since there are none, the argument is really just a vast question-begging fallacy.

The rest of your comment is likewise pathetic.

In other words, you have absolutely no frakking idea of what you are talking about. And then it is *me* that "did not bother trying to..."? Am I insulting you by registering the demonstrable fact that you do not know what you are talking about? If yes, then I suppose I am indeed insulting you.

Ilíon said...

B.Prokop: "If you can't do that, then you are either:

a) a liar, or
b) a fool

There is no third alternative.
"

'Fool' is a specialized species of 'liar'. A liar lies episodically, a fool lies systemically.

The three general logical alternative explanations for whay someone assets (or believe) a false thing are:
1) inability to understand the truth (i.e. stupidity)
2) (honest) misunderstanding of the truth (i.e. ignorance)
3) (dishonest) "misunderstanding" of the truth -- which covers disinclination to learn the truth to lying to intellectual dishonesty.

B. Prokop said...

Hmmm... I guess there was a third alternative after all. Thanks for clarifying that!

Hugo,

"Do you think that your mind, your consciousness exists regardless of what body you have, now and in the future, or do you think that you exist and are conscious because your material body exists? What you posted yesterday seems to point to the former"

You are correct. My posting of June 16, 2014 1:34 PM makes it (I hope) clear that I regard my current physical body as "mostly" irrelevant to the existence of "me" as a distinct and actual entity. I qualify that statement with a "mostly" because it is self-evident that my body can influence my consciousness (e.g., sleep, mental disorders, etc.). But my being is ultimately not dependent upon whether or not I am conscious. (I.e., I still exist while asleep. Unlike for a goose, it's not a new world every day. *)

This is yet another reason to admire the astonishing wisdom of Catholic doctrine, where it insists that Man is a hybrid (?composite? - not sure of the technical term here) creature composed of body and soul. We are neither angels (pure intellect) nor brutes (pure matter). Note that this teaching allows for physical processes to interact with spiritual ones, and for the two to affect one another. No conflict there!

* Old Russian Insult: "He's like a goose. It's a new world every morning.")

grodrigues said...

@B. Prokop:

"This is yet another reason to admire the astonishing wisdom of Catholic doctrine, where it insists that Man is a hybrid (?composite? - not sure of the technical term here) creature composed of body and soul."

Composite.

From the CC:

365 The unity of soul and body is so profound that one has to consider the soul to be the "form" of the body:234 i.e., it is because of its spiritual soul that the body made of matter becomes a living, human body; spirit and matter, in man, are not two natures united, but rather their union forms a single nature.

234 is a footnote: Cf. Council of Vienne (1312): DS 902.

B. Prokop said...

Thanks. I agree that it's important to use the right words (which is why I telegraphed my uncertainty there).

B. Prokop said...

"spirit and matter, in man, are not two natures united, but rather their union forms a single nature."

I very much like that wording. Says much more clearly what I was trying to express above, where I wrote that within an individual human being, physical processes interact with spiritual ones, and the two will affect one another. (or words to that effect)

World of Facts said...

grodrigues said...
"So these two principles can only be defended by philosophers with prior commitments antithetical to physicalism?"

Nope, my mistake, I did not intend to imply that if that's how it sounded. You can forget that part.

Going back to Victor's quote that you posted: "If a physical account of the process is causally complete, and physics is mechanistic, how do reasons come into play? "
Human beings are physical and capable of reason. So yes, the "reason-er" is someone who has a physical body and think about things. Right away, that's where I fail to see how starting from reason and going back to denying its physicality supports the notion that there exist non-physical things independently of the real physical world we inhabit. It does work if you start with the primacy of consciousness; defining certain things to exist non-materially first.

"All versions of the AfR are a deductive *argument* that purport to show that rational inference cannot be wholly reduced to the physical facts."

I understand that part very well yes. And my point is that, of course, you cannot reduce rational inference to physical facts when you imply that reason exists non-materially first. It would contradict that initial assumption.

"Let us continue with your scholarly and erudite display of knowledge.
[...]
you show a complete ignorance of what those arguments *actually* are and conclude, based on your ignorance, that since there are none, the argument is really just a vast question-begging fallacy.

The rest of your comment is likewise pathetic.

In other words, you have absolutely no frakking idea of what you are talking about. And then it is *me* that "did not bother trying to..."? Am I insulting you by registering the demonstrable fact that you do not know what you are talking about? If yes, then I suppose I am indeed insulting you.
"

I like how you 'need' (why?) to insert these commentaries. We are just people talking, sharing opinions mostly as neither of us has some sort of exclusive grasp on truth. The problem I had with your previous comment is that you did not bother specifying what sentences you were addressing, nor even a specific idea I hold on to. This is exactly the same thing William just did and I will reply to that next. This time though, you did quote me so it's much better (though you couldn’t help but have half your comment be 'commentaries' on how wrong and ignorant you think I am), and I even changed 1 of my sentences to continue that discussion. In other words, feel free to keep writing useless sentences like these, I will just ignore from now on and address only the actual content of your comments.

World of Facts said...

William said...
"you claim that all aspects of human experience are physical, because bodies are physical. This is a proof by definition"

No you seem to be over-simplifying by equating 'are physical' with 'depend on the physical', or 'relate to the physical'. Ironically, I also keep insisting that the AfR and other argument for non-material existence are guilty of 'proof by definition'. Let me quote some of your sentences:

- "If we define the mental as physical, then all we have left is a causal chasm between the microphysical properties (which do not determine much else of the putatively physical) and everything else."
So we must 'define' the mental as non-physical?

-"All dualists are doing is defining a subset of what you think to be physical as non-physical. "
So we must 'define' that subset as non-physical and conclude that non-physical existence exists regardless of the base physical reality we live in?

-"separate the physcial aspects of humans that include things like meaning, thought, feeling from the rest of what is physical: let's call them "sphysical" instead."
So we must 'define' these things as non-physical, or sphysical, regardless of the fact that ehy are 'mental' things physical human beings depict?

- "Have we changed anything in your view? Not necessarily, just redefined part of the physical.
Now let us further say the the "sphysical" is NOT physical, by OUR definition.
"
So 1 last time, let's define certain things as non-physical because they just are, regardless of their origin, meaning, purpose, broader definition.

No, nothing changed in my view. I start from the physical world to determine what exists or not. Within that world, there are humans who can think. Thinking is a physical process that we both understand very well, and don't, at the same time (to the risk of sounding self-contradictory).

We understand very well that thinking is all about having a mental representation of the real physical world "in our head". Essentially, thinking is always about remembering parts of the real physical experience we have and re-organizing it to form a conscious image. The hard part, the complex part, which we cannot fully explain is 'how' this process happens exactly. This still poses no problem to the physical nature of the process, nor does it make the conclusion that some literally non-material things exist.

In other words, it's not because we label certain things as 'mental', 'abstract', 'conceptual' vs 'physical' or 'material' that we instantly contradict the notion that all there is physical/material/natural. The charge of 'proof by definition' is, ironically, precisely what I think the dualists are doing.

World of Facts said...

@William, cont.
Let me try this again this way, using the 3 examples you gave: meaning, thought, feeling. As you read these lines, 'you', William, or whatever your real name is if it's not William, have both a physical and mental experience. I can know, for sure, that your eyes are bombarded by photons from the screen you are using, which trigger electrical signals to your brain. I can also know, for sure, that you experience something 'mental'. Your brain processes the electrical impulses and generates a conscious experience. The brain activity could be detected physically but the exact nature of your conscious experience cannot. Yet, it's not as if it's completely mysterious either; it is actually quite easy to know the limitation of your conscious experience.

Because our physical reality comes first, I know that your conscious experience is nothing more than the sum of some prior physical experience. No matter what you think about when reading these words, even though I cannot know exactly how it feels for you, I can know that something purely physical is happening. Your brain makes you process words written in English with some pre-defined meaning embedded in the words by generations of English speakers. The 'meaning' might be 'non-physical' or 'sphysical' in the sense that it's not something we can weigh, measure or count, but it spans from the material experiences we share now, and that others shared before us. In that sense, it all comes back to the material world as the basis for existence and provide a solid framework to explain and understand the real world we live in.

To continue, let me quote Mr. Prokop and answer his comment, which you may or may not agree with, I don't know:
"You are correct. My posting of June 16, 2014 1:34 PM makes it (I hope) clear that I regard my current physical body as "mostly" irrelevant to the existence of "me" as a distinct and actual entity. I qualify that statement with a "mostly" because it is self-evident that my body can influence my consciousness (e.g., sleep, mental disorders, etc.). But mybeing is ultimately not dependent upon whether or not I am conscious. (I.e., I still exist while asleep. Unlike for a goose, it's not a new world every day. *)"
This is to me a complete reverse of the order of things, but an honest one. The primacy of consciousness expressed here makes it clear that the non-material mind of the thinker is what comes first. The mind is assumed to exist, regardless of the material body, and can then justify a belief in that body through mental observation and deduction. The body is thus proven to exist, rightly so, but not the mind. In my view, the body is assumes to exist and the mind exists as a consequence.

The 'I still exist while asleep' phrase could indeed be an argument in favor of the primacy of consciousness, if it were not disregarding the way the physical brain works. Deprive the brain from oxygen and there will be no more 'I'. Sleeping maintains the state of the brain, making it possible for the 'I' to survive sleep. Memories are conserved, physically, in the brain. Again, I cannot explain how each electrons are aligned to do so, but I do know that without a physical body, I cannot identify a conscious human being. I don't buy the idea that I am who I am regardless of my body; without my body I am not someone, I am nothing.

Let me know what you think, pun intended ;)

And by the way, no need to compare what you think I think with skep's position. I may agree with him on many things, but I barely know him or his positions so it means little...

jdhuey said...

"spirit and matter, in man, are not two natures united, but rather their union forms a single nature."

I would say: "As far as we know, matter alone in it's various forms and nature forms man's multi-faceted nature."

William said...

Hugo,

I find your discourse on brain function boring; we knew more than that 40 years ago.

Please define what is physical, and give examples of something in human experience that is not physical, if any.

If you do so, I might be able to understand why you may think your use of "physical" is not too broad.

World of Facts said...

William,
"I find your discourse on brain function boring; we knew more than that 40 years ago."
Funny, as if I was trying to explain something controversial or new about the brain... So we agree about how it works, great.

"Please define what is physical, and give examples of something in human experience that is not physical, if any."
You, Will, you are physical. You have a body and a brain. Now, I am asking you this: think about the number 3, think about the number 4, now do the addition in your head. Did you reach 7? Great, you just did something non-physical by the everyday usage of the term non-physical.

Now, explain to me how you could have done that without being a physical human being?

Explain to me why I should conclude (if that's what you think) that there is such a thing as a literal non-physical existence, completely independent of the real physical world, simply because I just agreed with you that we do label things as non-physical when referring to our mental abilities and/or labels associated with the real world?

grodrigues said...

@Hugo:

"And my point is that, of course, you cannot reduce rational inference to physical facts when you imply that reason exists non-materially first. It would contradict that initial assumption."

And my point, for the umpteenth time, is that *nowhere* such an assumption is made, rather it is the *conclusion* of the argument, whose general logical structure is that of a reductio. So effectively, what you are saying is that a question begging fallacy is being committed. But you just assert it, you do not argue it. You do not even show the *least* awareness of how the conclusion is argued for. In the Blackwell article, Reppert discusses 6 subarguments to sustain his main thesis:

1. The argument from intentionality

2. The argument from truth

3. The argument from mental causation in virtue of propositional content

4. The argument from the psychological relevance of logical laws

5. The argument from the unity of consciousness

6. The argument from the reliability of our rational faculties

So you have to go through them and actually *show* that question-begging is going on, and where exactly it is going on.

Furthermore, as if bravado on top of ignorance were not enough, you go on to do *exactly* what you accuse Reppert of doing. From June 17, 2014 8:59 PM:

"But if we start with the physical world as a basis, assume that we exist in it as conscious physical beings; then it is possible that this conscious experience is nothing more than the result of the physical body."

So if we assume that we are wholly physical beings it is possible that we are wholly physical beings? No shit, Sherlock? So if Reppert starts with the assumption "that reason exists non-materially first" (which he does *not*) and then goes on to derive that it is impossible to "reduce rational inference to physical facts" that is a major crime, but if it is you doing it, as you yourself plainly state it, that is what?

"We are just people talking, sharing opinions mostly as neither of us has some sort of exclusive grasp on truth."

I will grant you that you are venting your opinions, alright.

William said...

Very good, Hugo, so you would say that thought of a mathematical object is not physical, by definition?

Now: if you say thoughts actually exist, this is a form of dualism, at least about abstract objects. If you say thoughts do not actually exist, the argument from reason applies to your position, rendering it incoherent.

William said...

Hugo:

I don't think that we need to say that non-physical things are independent of physical things for the argument from reason to work, so I am not trying to agree with your strawman position, nor defend it!

One issue with Victor's version of the argument from reason is that it depends on there being both physical and non-physical things, even if the two are NOT independent of each other.

The argument from reason be undermined by defining everything to be physical, though only in a superficial way (because it simply changes the argument to one of terminology).

It was that definition I was trying to clarify.

World of Facts said...

@ grodrigues
I am attempting to address specific arguments; sorry if this is not direct enough for you. Which one of the 6 should we look at more precisely? Because again, if you quote a book, I can quote books too.

Next, I said:
"But if we start with the physical world as a basis, assume that we exist in it as conscious physical beings; then it is possible that this conscious experience is nothing more than the result of the physical body."

You replied:
" So if we assume that we are wholly physical beings it is possible that we are wholly physical beings?"

No, you are jumping ahead. The assumption is that we exist as physical beings, nothing more. Then, we observe that we, humans, have what we call 'consciousness' and share that experience with each other, trying to understand how it works, what its limitations are, and so on. Since we started with the physical, it's possible that this is all there is, that nothing else affects that physical body we possess.

@ William
" I don't think that we need to say that non-physical things are independent of physical things for the argument from reason to work"

That's interesting. Perhaps I misunderstand that part of the argument but it seems to me that if non-physical things are not independent of physical things, then 'reason' can be dependent on the physical and thus the AfR fails, since:
"All versions of the AfR are a deductive *argument* that purport to show that rational inference cannot be wholly reduced to the physical facts."

" you would say that thought of a mathematical object is not physical, by definition? "

We don't describe the thought as a physical thing because we take a shortcut and define it in a more convenient way by contrasting the mental with the physical. But my position is not that the mental is non-existent, but that it exists because the physical exists. So the thought is physical in that sense, though it's useless to label it as physical in everyday usage.

I think people use an analogy with water sometimes, where atoms of H2O are not 'wet' yet their combination is. The thought is to the brain as water is to the molecules. The thought is non-physical yet the brain is physical; water is wet yet atoms are not. Not a perfect analogy but that's how I see it.

Will you answer my questions?

William said...

To answer yuor questions, Hugo:

"
Now, explain to me how you could have done that without being a physical human being? "
---
I cannot even explain how I would do that AS a physical human being, so this is beside the point! Perhaps you can explain how how do that as a purely physcial being :-) ?

"
Explain to me why I should conclude (if that's what you think) that there is such a thing as a literal non-physical existence, completely independent of the real physical world, simply because I just agreed with you that we do label things as non-physical when referring to our mental abilities and/or labels associated with the real world?"
----
I don't think that you need to conclude that, so I have no need to explain why you should conclude that, Mr. Strawman!


World of Facts said...

"I cannot even explain how I would do that AS a physical human being, so this is beside the point! Perhaps you can explain how how do that as a purely physcial being :-) ?"

You're a living human who you can think, that's it.
What's non physical about this?

"
I don't think that you need to conclude that"

'That' was the notion that there is an independent non-material existence. So fair enough, I will accept your adive and stick with matetialism, where material existence is a basic reality, without any evidence for something more to it. Thanks for confirming my views are justified ;-)

grodrigues said...

@Hugo:

"I am attempting to address specific arguments; sorry if this is not direct enough for you. Which one of the 6 should we look at more precisely? Because again, if you quote a book, I can quote books too."

You in June 17, 2014 8:43 AM: "Show me your favorite version and I will point out to you where it fails because of this assumption."

Me in June 17, 2014 9:11 AM: "So let's start here. Tell us where the argument goes wrong with quotes and page number references."

Then in June 17, 2014 9:41 AM after gracing us with a link to Philosopher Extraordinaire Richard Carrier, you point out what the alleged failure:

"This premise is accepted because of the notion that minds are assumed to be non-material. It's indirectly implied by contrasting 'rationally inferred' with 'nonrational causes'."

So you, with Carrier, claim that Reppert is begging the question. And you do this without quoting the argument or even address it.

Then I responded in June 17, 2014 10:27 AM with a quote from a response of Reppert to Carrier, that shows that the alleged fallacy of question-begging is not being committed.

Then in June 17, 2014 8:59 PM you address this last quote. In June 18, 2014 3:08 AM, I address your June 17, 2014 8:59 PM comment and *show* that it is a complete mess, betraying a total lack of awareness of what the argument actually is.

In June 18, 2014 11:48 AM, while acknowledging the blunder (thank you, as the effort was not all in vain), you go on to merely repeat the claim of question begging, do not address *any* of the arguments Reppert gives and do not even show the least awareness that such arguments exist e.g. "Right away, that's where I fail to see how starting from reason and going back to denying its physicality [...]" How can you "see" anything whatsoever if you are ignorant of the arguments used to substantiate the conclusion?

In June 18, 2014 1:03 PM, I point this out, this time even expliciting the subarguments Reppert uses to substantiate the main thesis under dispute, and indirectly, that no question-begging fallacy is being committed.

Response: "I am attempting to address specific arguments; sorry if this is not direct enough for you. Which one of the 6 should we look at more precisely? Because again, if you quote a book, I can quote books too." followed by more of the same piffle.

Shrug shoulders.

B. Prokop said...
This comment has been removed by the author.
B. Prokop said...

"Since we started with the physical, it's possible that this is all there is, that nothing else affects that physical body we possess."

Is that as far as you are going to take that argument? If so, it is very difficult (and rather pointless) to dispute it. Because many things are "possible" in the opening stages of an argument. But once we proceed from our going-ins, the possibilities rapidly diminish.

You see, you started with the physical, but immediately introduced consciousness, without in any way determining whether or not this new factor was also physical.

The mere fact that our physical bodies and our consciousness interact does not in any way lead to the conclusion that they are part and parcel of each other, or even things of the same nature. A sailboat interacts with the wind, but the two entities are quite different things.

I could re-write your opening premise as follows: "We start with the physical. Yet our existence within this physical world is as conscious physical beings. Might I say that it is even exclusively as conscious beings that we relate to the physical world at all. It is therefore possible that our conscious experience is something that interacts with, but is not part of, our physical body."

(Above posting edited to clear up potentially misleading sentence.)

William said...

Hugo,

there is a difference between saying that a thing is made of matter and saying that its matter is all there is to it. If you cannot grasp that, you are stupid, and I do NOT think you are stupid. You just want to stay safely within your worldview, I suppose.

World of Facts said...

@ grodrigues
"claim that Reppert is begging the question. And you do this without quoting the argument or even address it."

Every time I ran into arguments about non-material existence that's what it boils down to. I could be wrong... but you don't show anything. The quote I addressed above did show some question begging imo. You say it does not. Ok, can we move on to another piece that may be easier to discuss then? Looks like the answer is no...

"How can you "see" anything whatsoever if you are ignorant of the arguments used to substantiate the conclusion?"
I am trying to address points which I think were made by Victor or others. Your response seems to be that I am wrong, ignorant but do not really explain why or what I got wrong. You just assert and include random insult and *sigh* here and there. I see no attempt whatsoever on your part to explain anything.

"Shrug shoulders. "
As I just said...

@ Mr. Prokop
"You see, you started with the physical, but immediately introduced consciousness, without in any way determining whether or not this new factor was also physical. "

More or less. Yes, my view is not that the physical is all there is by definition. Rather, the presence of my physical body in this real world shows that I exist, objectively. If my body did not exist, I would not exist as a conscious person. Why? Because it's my body that depicts the attributes of a conscious human being; not what I am thinking in my head alone. So when I introduce consciousness, I introduce something that can be observed, physically, but that is open to the notion that there could be something more to it, at least in theory.

Next, I can try to justify whether all that's happening in my head is the result of this physical body or not. 1 by 1 we can analyzed the ideas that you and I share, or not, and share thoughts using that common reality. You can have, and do have, this idea that 'you' is defined by what you are thinking about, and I can have the opposite view, 'I' is defined by my body. My personality, my actions, my feelings, are all related to my physical body, and can be altered physically by external stimuli, some powerful enough to completely change my personality or even annihilate it, so I conclude that there is probably nothing else. Super quick summary...

@William

I appreciate that; I don't think anyone is fundamentally stupid around here either... but I also don't see the point of 'I want to stay safely withing my worldview'. Please let's not forget that you guys are writing on a blog of someone who agrees with you, but I don't, so I am certainly not the one wanting to remain 'comfortable'...

Regarding your comment, of course, 'there is a difference between saying that a thing is made of matter and saying that its matter is all there is to it'. What's the point?

B. Prokop said...

'I' is defined by my body.

Way up above, when I said your conclusion was a result of your priors, this is what I was speaking of.

Also, you are engaging in argument by definition:

Step 1: Present the definition
'I' is defined by my body.


Step 2: Describe the relevant qualities of X.
My personality, my actions, my feelings, are all related to my physical body, and can be altered physically by external stimuli...

Step 3: Show how X meets (or fails to meet) the definition
You actually skipped * this step.

Step 4: Conclude that X belongs within that class (or does not belong within that class).
so I conclude that there is probably nothing else.

* Hah! I originally typoed this line as "You actually skepped this step." Freudian slip, I guess!

grodrigues said...

@Hugo:

"As I just said..."

At least we agree on one thing: this is going nowhere.

There may still be hope for the two of us.

David Duffy said...

@Hugo:

" 'As I just said...'"

"At least we agree on one thing: this is going nowhere."

As an outsider (in my education and everyday conversions in life) reading through the comments, I didn't think the comments were going nowhere. I found the thoughts interesting. The difficult part for me is trying to translate my understanding of certain words into your understanding.

Illion: I'm not familiar with placing a word between two asterisks. Could you explain that literary device?

World of Facts said...

@ grodrigues
Sure, I think we'll meet again on another thread. I spent enough time here already... but I agree with what Dave just wrote, it was not going nowhere. There is a lot of interesting things to discuss which relate directly or indirectly to the AfR and the incompatible Materialist worldview.

@ Mr. Prokop
I can play this game too :)

" Step 1: Present the definition"
" I know that my consciousness is real. I do not know that it is a physical entity. "

" Step 2: Describe the relevant qualities of X."
" I regard my current physical body as "mostly" irrelevant to the existence of "me" as a distinct and actual entity."

"Step 3: Show how X meets (or fails to meet) the definition"
" within an individual human being, physical processes interact with spiritual ones, and the two will affect one another."

" Step 4: Conclude that X belongs within that class (or does not belong within that class)."
"my being is ultimately not dependent upon whether or not I am conscious"

B. Prokop said...

Hugo,

It's clear you're watching the World Cup - I assume you're rooting for England.

Ilíon said...

"Illion [sic]: I'm not familiar with placing a word between two asterisks. Could you explain that literary device?"

It's one (very common) means of *stressing* a word. It's similar to, but I think cleaner than, using underscores to _stress_ a word. In some few contexts, one might use caps to HIGHLIGHT a word, but that is a different matter from stressing a word.

B. Prokop said...

How about _*THIS*_???

B. Prokop said...

By the way, is there any way to underline a word in these comments?

Ilíon said...

If you could underline a word in the Blogger commbox, it likely would be done like this: <u>underline</u>

Ilíon said...

William: "there is a difference between saying that a thing is made of matter and saying that its matter is all there is to it"

C S Lewis: "In our world," said Eustace, "a star is a huge ball of flaming gas."

"Even in your world, my son, that is not what a star is, but only what it is made of."


My house is made of wood, and iron, and stone ... and a bit of my blood ... but that is not what a house *is*, else any old pile of wood and iron and stone would be a house.

William: "... If you cannot grasp that, you are stupid, and I do NOT think you are stupid. You just want to stay safely within your worldview, I suppose."

As I pointed out above, there are three (and there are only these three) general categories of explanation for why someone believes or asserts a false proposition:
1) stupidity, the inability to understand the truth of the matter;
2) ignorance, the (honest) failure to correctly understand the truth of the matter;
3) dishonesty, the (dishonest) disinclination (of one sort or another) to understand and/or acknowledge the truth of the matter.

The hypothesis that these 'materialists' are, in general,too stupid to grasp their error isn't really plausible, however much that it is a logical possibility.

The hypothesis that these 'materialists' are simply too ignorant to grasp their error is ruled out but the fact that their errors have been explained, over and over again, and yet they persist in them, including persisting in misrepresenting all sorts of aspects of the whole matter.

The hypothesis that these 'materialists' are simply too dishonest to acknowledge-and-correct their error is supported by multiple lines, including (but not limited to):
1) that they *do* constantly misrepresent the issue(s), even after their misrepresentations are explained;
2) that they *do* "reason" illogically in support of their error, even after their flaws of reasoning have been explained;
2a) that they *do* constantly beg the question, even after it is shown that they are doing that;
2b) that they *do* assert both 'A' and 'not-A' in support of their error, sometimes simultaneously, even after it is shown that they are doing that (one example: mocking Biblical miracles ... while asserting that one's car may spontaneously "ooze" through the garage wall and park itself on the street);
2z) that they characterize any act of reasoning as "logical" by its conclusion, rather than by its form or structure in accordance with the well-known rules of logical reasoning -- that is, if it can be used to support 'materialism', them it is "logical", but if it can be used to support 'anti-materialism' then it is "illogical";
3) that they *do* retreat into open irrationality so as to protect their error when the strain of deflecting the sound logic that illuminated their error becomes too great a burden;
3a) that they *do* deny their own existence rather than simply acknowledge that God is;

im-skeptical said...

4) that they base their beliefs on evidence;

B. Prokop said...

"that they base their beliefs on evidence"

Ha, ha, ha, HA, HA, HA-A-A-A-A !!!!!

This from the guy who, despite endlessly repeating "The evidence is out there!" cannot come up with the least, tiniest, most insignificant shred of evidence for his own statements. How 'bout it, Skep?

- Identify the portion of scripture that has been modified to comply with Church dogma (chapter and verse, please).

- Give us the "before" version, so we can confirm that it has actually been modified.

- Explain what dogma said modification is being complied with.


Still can't do it, huh? And you have the laughable nerve to say "they [meaning, of course, you] base their beliefs on evidence."

Gimme a break!

B. Prokop said...

"they base their beliefs on evidence"

By the way, Skep, did you even read Ilion's posting? Here is what he said (emphasis added):

"there are three (and there are only these three) general categories of explanation for why someone believes or asserts a false proposition"

So you are telling us that there are people who believe in a false proposition based on evidence? Just how is that done?

Ilíon said...

I-pretend-to-be-coherent-and-rationally-consistent: "Amorbis' infinite regress is a direct result of his dualism. There has to be a little man inside the material brain that serves as the interpreter. But how does this homunculus understand? Well, it's the little man inside him ...

This is the absurdity of dualistic thinking - your a priori belief that understanding can ONLY be done by an immaterial little man inside us, which would end the regress. Dump your dualism. It's incoherent.
"

amorbis: "it follows that our brain processes only count as interpreting insofar as they are interpreted as interpreting ..."

I-pretend-to-be-coherent-and-rationally-consistent: "If that isn't dualist thinking, then there is no dualism. You should try to learn about how the mind works from a more scientific perspective."

I invite you, boys and girls, to contemplete this steaming pile offered up by I-pretend (and really, by *all* the deniers of the "primacy of mind", as one of them recently put it).

Consider:

1) on the one hand, logical reasoning establishes that God-denial is absurd, for it necessarily spawns countless absurdities ... but they will never acknowledge either that fact nor the fact that, ergo, God is.

2) on the other hand, this sad little fool as asserting that "Science!" and his little sister (*) "Logic!" have established that "dualism" (or, in general, "pluralism") is the false conception of reality -- that is: that all things that exist are made of the same type of substance, to wit: "matter".

BUT, where/what is the logic, or the science, that has demonstrated this silly-and-self-contradictory (as I'll get to) and sad little assertion that "matter is all there is"? There is no such logic, there is no act of reasoning that demonstrates the (baseless) assertion. Furthermore, and as the issue is totally outside the scope of real science, there never can be a scientific demonstration of the assertion.

So, there is no logical reasoning, anywhere, that has ever established that "monism" -- much less materialistic monism -- is the correct view of the ultimate nature of reality. Meanwhile, our common lived experience certainly more than suggests that there is a plurality of substances making up reality.

What is that silly (and tendentious) assertion of which 'Science!' fetishists are so fond? "Extraordinary claims require extraordinary evidence!" Well, where is the "extraordinary evidence" that our self-understanding, which flows from our common lived experience, is all wrong (as distinct from incomplete)? Where do *they* get off denegrating our knowledge that we really do exist as mere "folk psychology"?

(*) Now, in truth, logic is superior to (actual) science, as the very prospect of doing science depends upon reasoning logically, but 'Science!' fetishists always turn that around; such that no matter what the deliverances of sound-and-valid logical reasoning, if it contradicts their reductive-and-eliminative mechanical determinism (**)

(**) the main reason that 'materialists' assert 'materialism' *isn't* for its (definitional) assertion that "matter is all there is", but rather for its logical entailment that *all things* are mechanically deterministic. It's not so much that they love "matter" as that they hate agency and the entailment of liberty.

Ilíon said...

Concerning the silly-and-self-contradictory of sad little assertion of "matter is all there is" monism --

Consder -- the term 'matter' refers to a concept, a category, a universal, not to any materially/physically concrete thing: you can't go to your local 'Science!' supply store and but five pounds of "matter", for there is no such thing as mere matter.

SO, even by their own twisted "reasoning", materialistic monism is self-contradictory, for there is (under their terminolgy) no such thing as "matter", but rather a plurality of types of "matter".

Now, 'matter' refers to a categorical concept. BUT, 'matter' is the sort of concept that can exist only if there is at least one other category in contrast to it.

Consider -- if the only species of animals in our experience were mammals, we wouldn't (and couldn't) even *have* the concept 'mammal'. And, if some novelist were to write some speculative fiction involving "animals that aren't born alive and don't (and can't) suckle their young", his readers would tend to understand him to be talking about "animals that aren't animals"; that is, they would understand it as highest non-sense.

There is no "mammal" that is simply a mammal: all "mammals" are of some specific type, some 'species' or other, of mammal.

Likewise, there is no "matter" that is simply matter: all "bits of matter" are of some specific type, some "species" or other, of matter. And this plurality extends sub-atomically.

B. Prokop said...

Or... is it possible?... that you are admitting that your "evidence-based" beliefs are... (wait for it)... false???

Ilíon said...

*grin*

im-skeptical said...

Bob, you miss the point completely (what a surprise). This addendum to Ilion's line of reasoning is entirely consistent with the rest of his wacky pronouncements. That basing your beliefs on evidence instead of religious faith is just as absurd as all the other attributes he hangs on materialists.

Shackleman said...

im-skeptical said... "4) that they base their beliefs on evidence;

Sorry to throw my hat in the ring here...but I grow very tired, and irritated when I hear this over and over especially because it's so untrue in most cases, and comes frequently with such snobbery.

You most certainly do *not* base your beliefs on "evidence" unless you yourself have gathered the evidence or you yourself have performed the experiments. Worse, you even *deny* the evidence of your very own experiences! Instead, you base your beliefs, almost entirely on testimony. Testimony of others who claim to have gathered the evidence.

Reading books ABOUT evidence, is not evidence.

What you should have said is:
"4) that they base their beliefs on the testimony of people who claim to be scientists"

im-skeptical said...

"Reading books ABOUT evidence, is not evidence."

So it is Shackleman's contention that Christians have absolutely no evidence to believe the myths about the life and resurrection of Jesus. I'm sure they will concur with that.

Shackleman said...

im-skeptical,

I made no such contention. Just pointing out your hypocrisy, is all.

What you and others on this thread believe in, is *testimony*, *despite* the evidence of your own experiences as a person with a mind and consious. You deny this first-hand evidence of your self, in favor of the testimony of other meat machines.

That's all good...but then the hubris with which you write isn't at all justified, now is it?

Further, you don't seem to realize that what you "believe" in wouldn't pass *your* standard for evidence. Again, you believe someone else's testimony of the evidence. You yourself have not seen the evidence.

Just admit it...90% or more of what you believe is based on testimony, and testimony alone.

im-skeptical said...

"I made no such contention."
- I think you did. You said that a book doesn't count as evidence. Christians rely on the NT for their 'evidence' of the Jesus story. What else do you have?

"What you and others on this thread believe in, is *testimony*, *despite* the evidence of your own experiences"
- Let's get this straight. I believe BECAUSE of my own experience that I am a material creature with mind and consciousness. I don't see any ghosts floating around, do you? No little man living inside me. All I have real evidence of is the material me. You, on the other hand, have been told that there IS a little man inside you (a soul, I think you call it). And this soul does your thinking and reasoning for you. You believe it's true, but what evidence do you have?

"That's all good...but then the hubris with which you write isn't at all justified, now is it?"

"what you "believe" in wouldn't pass *your* standard for evidence. Again, you believe someone else's testimony of the evidence. You yourself have not seen the evidence. "
- You don't know what evidence I've seen. You don't know what my education and professional experience are. You make some pretty bold assumptions about me. What is your evidence? And futhermore, you don't seem to understand what evidence is. Can we not rely on the foundation of knowledge that has been built up by those who came before us? It's something that even the greatest scientists and philosophers do. But I would agree that merely being written in a book does not make it valid evidence. You have to ask yourself what is the source, whether it is reliable, whether there is sufficient documentation and corroborating information, can it be verified?

"Just admit it...90% or more of what you believe is based on testimony, and testimony alone."
- Sorry. I think you're wrong, although it might be true in your case. That doesn't mean it's true of me.


amorbis said...

You, on the other hand, have been told that there IS a little man inside you (a soul, I think you call it). And this soul does your thinking and reasoning for you. You believe it's true, but what evidence do you have?

Nope. Wrong. Incorrect. Untrue. What dualists actually believe is that all of the contents of consciousness - thoughts, feelings, mental images, sensations - are immaterial, and that they together constitute an immaterial mind/soul. We believe this because these contents of consciousness cannot be publically observed in the physical brain (although they can be indirectly inferred by observing brain activity and behavior), and because there are very powerful arguments to the effect that no combination of physical facts could possibly entail that they exist. We reject functionalist and behaviorist conceptions of the mind according to which the mind is simply a set of actions or abilities, and instead take the mind to be what introspection shows it to be - a stream of consciously experienced mental states. We do not believe in a "little man" inside us who "does our thinking and reasoning for us"; what we believe in is thinking and reasoning themselves as states of a stream of conscious experience that we call the mind or soul. You may disagree with this conception of the mind - perhaps you believe (erroneously) that cognitive science has proven it wrong - but to straw man it as something it is clearly not is simply intellectual dishonesty at its finest.

Shackleman said...

1007"You said that a book doesn't count as evidence. Christians rely on the NT for their 'evidence'..."

Let me be clear, I hold that testimony *is* evidence. But I've been around this block long enough to know, testimony is not the sort of evidence you're implying with your quip about beliefs. You're meaning evidence in the scientific, physical sense and you (collective you) routinely discount testimonial evidence. And the point you fail to actually attend to is *most* of your "beliefs" come to you via testimony alone.

"Let's get this straight. I believe BECAUSE of my own experience that I am a material creature with mind and consciousness...All I have real evidence of is the material me. "

im-skeptical, there *is* no "you" that is having "real experiences". "You" are a flesh bag of chemical goo. What you think is "you" is an illusion caused by your brain's chemical processes. So, if it's true that your sense of self, is but a byproduct of your brain's chemistry, then why do you trust your own experiences? This is all AfR 101, man.

"You, on the other hand, have been told that there IS a little man inside you (a soul, I think you call it). And this soul does your thinking and reasoning for you. You believe it's true, but what evidence do you have? "

Here, maybe this will help...Let's get this straight. I believe BECAUSE of my own experience that I am an immaterial soul with mind and consciousness....All I have real evidence of is the immaterial me.

Gee, that was easy.

"You don't know what evidence I've seen. You don't know what my education and professional experience are. You make some pretty bold assumptions about me. What is your evidence? And furthermore, you don't seem to understand what evidence is."

I wouldn't care if you were a Nobel prize winner in physics. You STILL wouldn't have seen first hand evidence for 90% or more of what you believe! Of all of your "evidence fueled beliefs", how many come to you first hand? As CS Lewis has said "The ordinary man believes in the Solar System, atoms, evolution, and the circulation of the blood on authority--because the scientists say so. Every historical statement in the world is believed on authority. none of us has seen the Norman Conquest or the defeat of the Spanish Armada. None of us could prove them by pure logic as you prove a thing in mathematics. We believe them simply because people who did see them have left writings that tell us about them: in fact, on authority".

I'm not claiming any special perch here...you are. Most of my knowledge comes to me via testimony. So does yours. You just don't, or *won't* see it.

"Can we not rely on the foundation of knowledge that has been built up by those who came before us?"

Sure, if you want. But then you're shifting the goalposts away from evidence and toward authority. You'll be defeating your own prior statements if you do so. I say, have at it.

"It's something that even the greatest scientists and philosophers do. But I would agree that merely being written in a book does not make it valid evidence. You have to ask yourself what is the source, whether it is reliable, whether there is sufficient documentation and corroborating information, can it be verified?".

That's cool, but then you're shifting goalposts again. Perhaps what you should have said was

"that they base their beliefs on the testimony of reliable witnesses"

But then, you wouldn't have much ammo against us Christians now would you?

Sorry. I think you're wrong, although it might be true in your case. That doesn't mean it's true of me."

Yes it is. You just don't or won't see and admit to it.

im-skeptical said...

We've been through all this before. You believe (without evidence) this little man, or soul, or whatever you want to call it exists separately from the physical body. You think that because you can't objectively observe subjective experience (which is by definition, subjective), that you can jump to the conclusion that it must be immaterial. You still don't have any evidence.

B. Prokop said...

"You still don't have any evidence."

Nor do you, apparently. If you did, you would have produced some of it before now, instead of just bleating "The evidence is out there!" without ever coughing any up.

Sheesh!

Shackleman said...

im-skeptical,

Why am I not surprised that instead of attending to my post, you just change the subject and fall back to attacking your straw man?


Carry on, I guess...

im-skeptical said...

Shackleman,

Do you or do you not believe you have a soul? Is this soul the thing that does your thinking? Is it an immaterial entity separate from your physical body? Is this a strawman, or is it what you believe?

B. Prokop said...

"Is [the soul] an immaterial entity separate from your physical body?"

I will thank grodrigues for citing The Catechism in this regard. I think it is the clearest statement possible on the subject:

"The unity of soul and body is so profound that one has to consider the soul to be the "form" of the body: i.e., it is because of its spiritual soul that the body made of matter becomes a living, human body; spirit and matter, in man, are not two natures united, but rather their union forms a single nature." (My Emphasis)

So no, the soul is NOT an immaterial entity separate from one's physical body. To say that it is, is very much a strawman.

amorbis said...

It's not a strawman, Bob - what the Catechism is referring to is what is known in philosophy as hylomorphic dualism (or Thomistic dualism), a form of dualism that is held by a minority of dualists living today. In essence, hylomorphic dualism treats the soul as the form of the body and treats all of the soul's powers except for intellectual activity (the grasping of abstract concepts and the ability to reason) as dependent on the body. (Since animals don't have intellectual activity, the implication is that animal souls are entirely dependent on matter and disappear when the animal dies. It also implies that we cannot have sensations, mental images, or emotions while disembodied.) Cartesian substance dualism, on the other hand, treats all mental phenomena (not just intellectual activity) as part of an immaterial entity separate from (but very closely connected with) the body.

B. Prokop said...

I'm not so sure that it's not a strawman. Just because there are people who embrace Cartesian substance dualism, doesn't make it mainstream, and it certainly does not appear to be orthodoxy. After all, we all know there are genuine YEC-ers out there, but YEC is still a strawman despite that fact.

Skep's question is at best a swing and a miss.

World of Facts said...

"we all know there are genuine YEC-ers out there, but YEC is still a strawman despite that fact"

What does that mean?

Ilíon said...

B.Prokop: "So no, the soul is NOT an immaterial entity separate from one's physical body. To say that it is, is very much a strawman."

But, of course one's soul or spirit is an immaterial entity separate (though, 'distinct' might be a better term) from one's physical body -- else there could be no resurrection of the individual person. Or, if one still isn't on-board with Christianity having the most correct understanding of reality, one could not *reason* if one were not separate or distinct from one's body.

The specific problem here is two-fold:

1) the God-hating reductive/eliminative materialists (not that there is any other kind) are setting up a strawman of non- or anti-[materialistic reductionism] ... despite that the error has been pointed out time and again;
2) you all are misunderstanding the strawman

*Look* again at what the petty fool keeps saying --

"We've been through all this before. You believe (without evidence) this little man, or soul, or whatever you want to call it exists separately from the physical body. You think that because you can't objectively observe subjective experience (which is by definition, subjective), that you can jump to the conclusion that it must be immaterial. You still don't have any evidence."

"Do you or do you not believe you have a soul? Is this soul the thing that does your thinking? Is it an immaterial entity separate from your physical body? Is this a strawman, or is it what you believe?"

He is presenting anti-[materialistic reductionism] as positing that there are three separate entities involved in the human person:
1) the person himself,
2) an immaterial soul or spirit that belongs to him, which does his immaterial thinking for him;
3) a material/physical body that belongs to him, which does his physical interacting with the material world for him.

But, of course, one's "immaterial soul or spirit" just *is* oneself. One has a body; but, one does not have a soul/spirit -- one *is* a soul/spirit.

Why do you think that I'm always saying that one doesn't "have" a free-will, but that one *is* a free-will; that one doesn't "have" a mind, but that one *is* a mind; that one doesn't "have" a self, but one *is* a self?

Ilíon said...

Hugo: "What does that mean?"

It means we're wandering into one of the topics concerning which B.Prokop puts on his intellectual dishonesty hat.

You should be able to sympathize with him. After all, your intellectual dishonesty hat is pretty ample.

Ilíon said...

Goodness! We haven't seen Shackleman in ages!

B. Prokop said...

"It means we're wandering into one of the topics concerning which B.Prokop puts on his intellectual dishonesty hat."

Hey, now that it's summer, it's time to switch out one's wardrobe. Gone are the furry hats with ear flaps, and out come the baseball caps!

"What does that mean?"

People (rightfully) accuse the presdent of using a strawman argument when he defends his policy in Syria by saying he doesn't want to send combat troops to that country. Despite the fact that there really are people out there who would like to see us do exactly that, it is still a strawman argument, because the people who actually count (i.e., his opponents in congress) are not among them.

im-skeptical said...

In answer to Ilion,

You make a good point, but I am not referring to three entities, as you surmise. The 'you' I speak of is whatever you think you are. For example, I speak of myself as having a physical body, but at the same time, I recognize that I am nothing more than that - my body is me.

"Skep's question is at best a swing and a miss."

Bullshit. Do you not believe that this soul lives on after the body dies? How can you then say that it is in unity with the body, when this is only a temporary affair? Clearly it has a separate existence, even in the view of Catholics.

You believe:
The soul is an immaterial entity.
The soul lives independently of the body (even if the body doesn't live independently of the soul).
The body has no ability to think or act without the soul.

This is no strawman. It's what you believe. And don't forget, I was raised as a Catholic. I was taught the Catechism, too.

amorbis said...

Just because there are people who embrace Cartesian substance dualism, doesn't make it mainstream, and it certainly does not appear to be orthodoxy. After all, we all know there are genuine YEC-ers out there, but YEC is still a strawman despite that fact.

Um, if anything, Cartesian substance dualism (or something very much like it, like emergent dualism) is MORE mainstream than Thomistic hylomorphic dualism; certainly, hylomorphic dualism is enough of a minority opinion that very few philosophers of mind even know it exists. And comparing substance dualism to YEC is downright insulting to philosophers like Richard Swinburne, William Hasker, Charles Taliaferro, and the vast majority of ordinary people (regardless of religion) who accept something at least something like substance dualism. Just because it's not Catholic orthodoxy does not make it a lunatic fringe opinion unworthy of consideration.

amorbis said...

Also, embracing substance dualism does not commit oneself to the view that the mind/soul is completely separate from the body. As Charles Taliaferro put it:

But what I do want
to stress at the end of this chapter is that materialist critics often exaggerate
and caricature dualism as holding that the soul- body relationship
must be like the relationship of someone (the soul) and their container
or vehicle or (as Dennett has proposed), the person is a tiny subject in
a theatre located in the head. Must dualism land us with the idea that
the body is like a mask or machine that a nonphysical ghost- like soul
controls? I have argued elsewhere that it does not.25 In a healthy, fully
functioning human being there need be no bifurcation of person and
body. To see me writing is not to see a soul controlling a body; it is to see
an embodied person. Dualism is best seen (in my view) along integrated
lines. In healthy conditions, mind and body, the mental and physical,
function as a unit. But under different conditions, in the case of death,
for example, the body may perish but, if a person is more than his or her
body, death may not mark the end of the person or soul.
(The Soul Hypothesis, p. 40)

B. Prokop said...

"Do you not believe that this soul lives on after the body dies? How can you then say that it is in unity with the body, when this is only a temporary affair? ... And don't forget, I was raised as a Catholic. I was taught the Catechism, too."

Well then, apparently none of it sank in. If it had, you'd know that the separation of the soul and body at death is unnatural, and a product of sin. It was one of the (if not the main) consequences of the fall of man. ("Out of [the ground] you were taken; you are dust, and to dust you shall return.")

So no, no, no! The separation of soul from body at death in no way implies that the two are not in unity. One might as well say that the handle is not in unity with the coffee cup, simply because it breaks off when dropped.

Ilíon said...

amorbis: "[@ June 21, 2014 9:05 AM]"

amorbis: "[@ June 21, 2014 9:11 AM]"

Other than implicitly agreeing with B.Prokop's denegration of YEC (*) as being as dispretutable as Darwinism, exactly.

(*) It's one thing to fault them if they resort to faulty reasoning; it's quite another to fault them because they dispute Darwinism or because they reject certain questionable scientistic assumptions or because one doesn't care for the conclusions at which they arrive having rejecting questionable scientistic assumptions.

B. Prokop said...

Let's get one thing perfectly clear here. I do not accuse YEC-ers of using faulty reasoning. I think their conclusions are perfectly reasonable, given their priors. What I disagree with are their going-in positions. Namely, that the Old Testament is word-for-word historically accurate. This notion, I believe, either totally ignores or flat out contradicts the varied and various literary types one finds in scripture. Yes, there is straight history in the OT. But there are also parables, allegory, epic poetry, myth (not a Bad Word in my lexicon), out-and-out fiction (again, not a Bad Word by any means), legal documents, family history (quite a different thing from "History"), letters, symbolic imagery, and probably a few more that I failed to mention (plus a few parts that defy categorization). After all, The Bible isn't even really a book - it's a library. Asking "Is the Bible literally true?" is like asking "Is the library literally true?"

It is my disagreement with the YEC-er's initial statements in their arguments that causes me to reject their conclusions - not the process by which they arrived at them.

Ilíon said...

B.Prokop: "So no, no, no!"

Too bad one can't underline. Double. With qoute marks, and three (!!!) exclamations!

B.Prokop: "The separation of soul from body at death in no way implies that the two are not in unity. One might as well say that the handle is not in unity with the coffee cup, simply because it breaks off when dropped."

Perhaps you mean 'union', rather than 'unity' -- both for the cup-and-handle and for the body-and-self?

The whole reason that the handle can detach from the broken cup -- and the reason it can even be seen and spoken of as a distinct entity -- is that it is not the cup. While the unbroken cup and the handle may be in union, they are not a unity.

Likewise, the whole reason that the "soul" can detach from the dead body -- and the reason it can even be seen and spoken of as a distinct entity -- is that it is not the body. While the living body and the "soul" may be in union, they are not a unity.

As you yourself have pointed out recently, the body is constantly changing, both in the arrangement of the matter of which it is comprised and in the specific "bits" of matter of which it is comprised; there is no real continuity of the body, and no real identity in the body. It is one's (immaterial) identity -- which is an aspect of one's immaterial self/mind/spirit/soul -- that (temporarily, so for) imposes an appearance of continuity and identity on the body.

B. Prokop said...

But does not the soul change as well? For instance, as a result of either sin or repentance?

(I realize there is genuine debate amongst at least Catholic theologians on this very point.)

Papalinton said...

Now that we've traversed the theological playground of souls, disembodied minds, poltergeists [my word for those apparent entities that inhabit an ethereal netherworld capable of interfering with the laws of physics in the natural world], revelation, dualism, and the ostensible immateriality of feeling, emotions, thinking and other things that apparently have no physical cause, I draw back to the AfR.

The ubiquitous furphy that philosophers who do not permit their metaphysics to supervene on the natural world [naturalism/physicalism] is that the corpus of arguments offered, particularly through philosophy of the theological variety, are precisely that, .... arguments. Arguments can indeed be logical, can indeed be reasonable. But to be of any substantive value they must be grounded in evidence, fact, in testable certitude, and not the form of opinioned certitude that is so characteristic of religion-grounded abstraction Theology based philosophy does not meet the standard of testability. The best such scholarship can hope to achieve is surmising, to suppose that something is true without having evidence to confirm it. In his new book Dr James A Lindsay, scientist and renowned mathematician eruditely captures the quintessential difference between the tradition of theo-philosophical surmising and the contemporary desire for a more empirical evidence-based philosophy to underpin and guide social, cultural and community decision-making.

At another place Dr Lindsay notes: "All philosophy of religion (and theology, which it seeks to work with and upon) have is arguments. But notice that if they had real evidence, they would need no arguments. This hints that the arguments themselves are probably redundant--on the one hand, without evidence, they cannot prove anything about reality, and on the other hand, with evidence, they're unnecessary.
They tell us what's going on, though. God doesn't exist. "God" is a philosophical concept, an abstraction, an idea. This is all well and good except that the idea is reified by billions and acted upon as if it is a real thing that will one day judge the living and the dead, and in the meantime alternately has absolute dominion over the universe and no capacity to cap human free will (which probably doesn't even exist and may not even be coherent--another abstract idea, then). The disease of theology, which is an infection in both society and philosophy, with philosophy of religion being the primary locus of disease, is in refusing to admit that "God" is just a concept. Just a concept. Moreover, except in the ways people act upon concepts they hold, concepts don't do anything."


When we look back in maudlin retrospection, it is with amusement that we are able to discern the shape of innumerable barrows, the burial sites of old and weary gods with which humanity has grown tired, the mass grave of old gods we rightly call mythology. I am reminded of the astute observation of one Ralph Waldo Emerson, renowned poet, author, philosopher: "The religion of one age is the literary entertainment of the next."

Amen.


amorbis said...

Paps,

Logical positivism has been dead for decades. Get with the times, bro.

World of Facts said...

líon said...
"your intellectual dishonesty hat is pretty ample."

I would like to know what you refer to, honestly. Though can I even use that expression with you? Or do you think I am so dishonest that nothing can have any value? It's quite interesting as I would never run into such situation anywhere else but on an online blog where religious views are discussed. So if you are interested, please do explain why you think I am dishonest, and about what exactly? I really don't know what you are talking about... Dishonesty implies that I lie about something; something that I know is true and claim is false, or vice versa. What are those claims?

Cheers